Adult Health 1

Réussis tes devoirs et examens dès maintenant avec Quizwiz!

What are the primary participants in the immune system?

T- and B- cell lymphocytes

Which assessment should be completed if immune dysfunction is suspected in the neurosensory system?

Ataxia

Which of the following cell types are involved in humoral immunity?

B lymphocytes

The nurse should assess all possible causes of pruritus for a patient complaining of generalized pruritus. What does the nurse understand can be another cause for this condition?

end-stage kidney disease

A client undergoes an arthroscopy at the outpatient clinic. After the procedure, the nurse provides discharge teaching. Which response by the client indicates the need for further teaching?

"I should use my heating pad this evening to reduce some of the pain in my knee." Explanation: The client requires additional teaching if he states that he'll use a heating pad to reduce pain the evening of the procedure. The client shouldn't use heat at the procedure site during the first 24 hours because doing so may increase localized swelling. Ice is indicated during this time. Elevating the extremity helps reduce swelling. The client may experience some discomfort after the procedure for which the physician may order medication. Bruising and swelling are common after an arthroscopy.

While reading a client's chart, the nurse notices that the client is documented to have paresthesia. The nurse plans care for a client with

abnormal sensations. Explanation: Abnormal sensations, such as burning, tingling, and numbness, are referred to as paresthesias. The absence of muscle tone suggesting nerve damage is referred to as paralysis. A fasciculation is the involuntary twitch of muscle fibers. A muscle that holds no tone is referred to as flaccid.

A client is prescribed montelukast as part of a treatment plan for an allergic disorder. The nurse understands that this drug belongs to which class?

leukotriene receptor antagonist

Which orthopedic surgery is done to correct and align a fracture after surgical dissection and exposure of the fracture? Joint arthroplasty Total joint arthroplasty Open reduction Arthrodesis

Open reduction Explanation: An open reduction is the correction and alignment of the fracture after surgical dissection and exposure of the fracture. Arthrodesis is immobilizing fusion of a joint. A joint arthroplasty or replacement is the replacement of joint surfaces with metal or synthetic materials. A total joint arthroplasty is the replacement of both the articular surfaces within a joint with metal or synthetic materials.

A patient is complaining of severe itching that intensifies at night. The nurse decides to assess the skin using a magnifying glass and penlight to look for the "itch mite." What skin condition does the nurse anticipate finding?

Scabies

A nurse provides nutritional health teaching to an adult client who had two fractures in 1 year. Besides recommending supplemental calcium, the nurse suggests a high-calcium diet. What would the nurse recommend that the client increase intake of?

Yogurt and cheese. Explanation: Yogurt and cheese are excellent sources of calcium. The other choices are low-calcium foods.

hypotonic solution

a solution with an osmolality lower than that of serum

isotonic solution

a solution with the same osmolality as blood

While in a skilled nursing facility, a client contracts scabies, which is diagnosed the day after discharge. The client is living at her daughter's home with six other people. During her visit to the clinic, the client asks a staff nurse, "What should my family do?" The most accurate response from the nurse is:

"All family members need to be treated."

The nurse is admitting a client to the unit with a diagnosis of ataxia-telangiectasia. The nurse would recognize that the client is exhibiting telangiectasia when assessing the presence of what?

Vascular lesions caused by dilated blood vessels

Which assessment finding(s) are likely to cause noncompliance with antiretroviral treatment? Select all that apply.

Active substance abuse

A client is experiencing muscle weakness in the upper extremities. The client raises an arm above the head but then loses the ability to maintain the position. Muscular dystrophy is suspected. Which diagnostic test would evaluate muscle weakness or deterioration?

An electromyography Explanation: An electromyography tests the electrical potential of muscles and nerves leading to the muscles. It is done to evaluate muscle weakness or deterioration. A serum calcium test evaluates the calcium in the blood. An arthroscopy assesses changes in the joint. An MRI identifies abnormalities in the targeted area.

A client with rheumatoid arthritis (RA) reports feeling down and not able to sleep. Which approaches would the nurse anticipate being considered for this client? Select all that apply. Antidepressant therapy Physical therapy Referral to group therapy Occupational therapy Corticosteroid therapy

Antidepressant therapy Referral to group therapy

When an infection is bloodborne, the manifestations include which symptom?

Chills Explanation: Manifestations of bloodborne infection include chills, high fever, rapid pulse, and generalized malaise.

A woman infected with HIV comes into the clinic. What symptoms may be the focus of a medical complaint in women infected with HIV?

Gynecologic problems

What does the nurse understand will result if the patient has a deficiency in the normal level of complement?

Increased susceptibility to infection

Which substance may be used to lubricate a condom?

K-Y Jelly

A nurse is caring for an adolescent client who has burn wounds on her face and hands. Which of the following statements by the client indicates that she has adapted to her changed body image?

May I go with my family to the visitor's lounge? This statement demonstrates a positive self-image. The client is asking to visit with her family in a public setting.

Which type of immunity becomes active as a result of infection by a specific microorganism?

Naturally acquired active immunity

In which response do T lymphocytes have a role?

Transplant rejection

A nurse in a clinic is assessing a client who has AIDS and a significantly decreased CD4-T-cell count. The nurse should recognize that the client is at risk for developing which of the following infectious oral conditions? Halitosis Gingivitis Xerostomia Candidiasis

Candidiasis Although oral candidiasis can affect anyone, it occurs most often in infants, toddlers, older adults, and clients whose immune systems have been compromised by illness, such as AIDS, or medications

A nurse is caring for a client who is postoperative following a urinary diversion to treat bladder cancer. Which of the following interventions should the nurse include in the plan of care?

Change the collection pouch in the early morning The nurse should plan to change the urinary collection pouch in the early morning when urine output is reduced.

A nurse on assesses a client with dark skin and notes new purple-gray cast to the skin on the chest, back, and arms. Which priority nursing intervention should the nurse implement?

Check the client's oral temperature.

A dark-skinned firefighter is admitted to the emergency room with smoke inhalation. An assessment result indicates possible carbon monoxide poisoning. What is the indicator noted on the assessment?

Cherry red color to the nail beds, lips, and oral mucosa

A nurse is monitoring the client's progression of human immunodeficiency virus (HIV). What debilitating gastrointestinal condition found in up to 90% of all AIDS clients should the nurse be aware of?

Chronic diarrhea

A nurse is caring for a client who has a fractured tibia as a result of a fall. The client x-ray shows that the bone is splintered into several pieces around the shaft. The nurse should identify that the client has which of the following types of fractures?

Comminuted With a comminuted fracture, the impact fragments the bone into several pieces

Which term refers to the shaft of the long bone?

Diaphysis Explanation: The diaphysis is primarily cortical bone. An epiphysis is an end of a long bone. Lordosis refers to an increase in lumbar curvature of spine. Scoliosis refers to lateral curving of the spine.

A patient is experiencing an allergic reaction to a dose of penicillin. What should the nurse look for in the patient's initial assessment?

Dyspnea, bronchospasm, and/or laryngeal edema.

The client with a fractured left humerus reports dyspnea and chest pain. Pulse oximetry is 88%. Temperature is 100.2 degrees Fahrenheit (38.5 degrees Centigrade); heart rate is 110 beats per minute; respiratory rate is 32 breaths per minute. The nurse suspects the client is experiencing: Complex regional pain syndrome Compartment syndrome Fat embolism syndrome Delayed union

Fat embolism syndrome Explanation: The clinical manifestations described in the scenario are characteristic of fat embolism syndrome.

The nurse observes a client's fingernails have a concave shape. What laboratory studies should the nurse review?

Hemoglobin and hematocrit

Which term means a lack of one or more of the five immunoglobulins?

Hypogammaglobulinemia

A client develops a facial rash and urticaria after receiving penicillin. Which laboratory value does the nurse expect to be elevated?

IgE

A 34-year-old client is diagnosed with chronic hepatitis C. Testing reveals that the client is a candidate for treatment. The nurse anticipates that which therapy could be used to treat the client's condition?

Interferon

Which of the following information regarding the transmission of lice would the nurse identify as a myth?

Lice can jump from one individual to another.

Which procedure done for skin cancer conserves the most amount of normal tissue?

Mohs micrographic surgery

Which of the following presents with an onset of heel pain with the first steps of the morning? Morton's neuroma Hallux valgus Plantar fasciitis Ganglion

Plantar fasciitis Explanation: Plantar fasciitis, an inflammation of the foot-supporting fascia, present as an acute onset of heal pain experienced with the first steps in the morning. Hallux valgus (commonly called a bunion) is a deformity in which the great toe deviates laterally. Morton's neuroma is a swelling of the third (lateral) branch of the median plantar nerve. A ganglion, a collection of gelatinous material near the tendon sheaths and joints, appears as a round, firm compressible cystic swelling, usually on the dorsum of the wrist.

A client has been taking a decongestant for allergic rhinitis. During a follow-up visit, which finding suggests that the decongestant has been effective?

Reduced sneezing

Reproductive health education for women who are HIV-positive includes recommending which of the following contraceptives?

The female condom

To treat a client with acne vulgaris, the physician is most likely to order which topical agent for nightly application?

Tretinoin (retinoic acid [Retin-A])

Which term refers to a condition characterized by destruction of melanocytes in circumscribed areas of the skin?

Vitiligo

A nurse is assessing a client with possible osteoarthritis. What is the most significant risk factor for primary osteoarthritis?

age Explanation: Age is the most significant risk factor for developing primary osteoarthritis. Development of primary osteoarthritis is influenced by genetic, metabolic, mechanical, and chemical factors. Secondary osteoarthritis usually has identifiable precipitating events such as trauma.

Normal sodium concentration

135-145 mEq/L (extracellular) 10-14 mEq/L (intracellular)

When do most perinatal HIV infections occur?

After exposure during delivery

A client calls the clinic and asks the nurse if using oxymetazoline nasal spray would be all right to relieve the nasal congestion the client is experiencing due to seasonal allergies. What instructions should the nurse provide to the client to avoid complications?

Do not overuse the medication as rebound congestion can occur.

A client with suspected exposure to HIV has been tested with the enzyme-linked immunosorbent assay (ELISA) with positive results twice. The next step for the nurse to explain to the client for confirmation of the diagnosis is to perform a:

Western blot test for confirmation of diagnosis.

Production of melanin is controlled by a hormone secreted by which of the following?

hypothalamus

Which of the following pigments influences hair color?

melanin

Ataxia refers to

uncoordinated muscle movement

The nurse is caring for a client with herpes zoster. The nurse documents the lesions as

vesicles

When a patient has been diagnosed with scabies, if the infection has spread, family members may complain of pruritus within which time frame?

1 month

While taking the health history of a newly admitted client, the nurse reviews general lifestyle behaviors. What strategies would have a positive effect on the immune system?

Biofeedback, relaxation, and hypnosis

The nurse is administering intravenous vancomycin. What will the nurse initially assess the client for if an allergic reaction occurs?

Dyspnea, bronchospasm, and/or laryngeal edema.

What types of cells are the primary targets of the healthy immune system? Select all that apply. Infectious cells Foreign cells Cancerous cells Typical cells

Infectious cells Foreign cells Cancerous cells

A patient is visiting the physician to determine what type of allergy is causing a rash. What type of testing does the nurse anticipate the physician will schedule?

Patch test

A patient had hand surgery to correct a Dupuytren's contracture. What nursing intervention is a priority postoperatively? Changing the dressing Performing hourly neurovascular assessments for the first 24 hours Having the patient exercise the fingers to avoid future contractures Applying a cock-up splint and immobilization

Performing hourly neurovascular assessments for the first 24 hours Explanation: Hourly neurovascular assessment of the exposed fingers for the first 24 hours following surgery is essential for monitoring function of the nerves and perfusion.

The nurse notes that the client demonstrates generalized pallor and recognizes that this finding may be indicative of

anemia

Which material consists of a powder in water?

suspension

Red bone marrow produces which of the following? Select all that apply. Corticosteroids Estrogen Platelets Red blood cells (RBCs) White blood cells (WBCs)

Platelets Red blood cells (RBCs) White blood cells (WBCs)

A nurse is caring for a client whose wounds are covered with a heterograft dressing. In response to the client's questions about the dressing, the nurse explains that it is obtained from which of the following sources?

Pig skin Heterografts are obtained from an animal, usually a pig

What is the term for a lateral curving of the spine?

Scoliosis Explanation: Scoliosis is a lateral curving of the spine. Lordosis is an increase in the lumbar curvature of the spine. Diaphysis is the shaft of a long bone. Epiphysis is the end of a long bone. Reference:

Which allergic reaction is potentially life threatening?

angioedema

A nurse comes to the employee health center for evaluation and is diagnosed with allergic contact dermatitis related to latex. What manifestation would the nurse most likely exhibit?

blistering

The nurse is reviewing the client's medications. What antihistamines are contraindicated for a client with narrow-angle glaucoma? Select all that apply. fexofenadine diphenhydramine loratadine cetirizine brompheniramine

diphenhydramine loratadine cetirizine brompheniramine

The patient is advised to apply a suspension-type lotion to a dermatosis site. The nurse should advise the patient to apply the lotion how often to be effective?

every 3 hours

The nurse notes that a client has round red macules over the lower extremities. The nurse documents this finding as

petechiae

A client with AIDS has become forgetful with a limited attention span, decreased ability to concentrate, and delusional thinking. What condition is represented by these symptoms?

AIDS dementia complex (ADC)

A nurse is assessing a client who has a lesion on his skin. Which of the following findings is a clinical manifestation of a malignant melanoma?

Irregularly shaped lesion with blue tones Malignant melanomas are irregularly shaped and can be blue, red, or white in tone. They often occur on the client's upper back and lower legs

A patient has enlarged lymph nodes in his neck and a sore throat. This inflammatory response is an example of a cellular immune response whereby:

Lymphocytes migrate to areas of the lymph node

Which serum level indicates the rate of bone turnover?

Osteocalcin Explanation: Serum osteocalcin (bone GLA protein) indicates the rate of bone turnover. Serum myoglobin is assessed to evaluate muscle trauma. Serum enzyme levels of creatinine kinase and aspartate aminotransferase become elevated with muscle damage.

A client is diagnosed with osteomyelitis. This is most commonly caused by which of the following? Escherichia coli Staphylococcus aureus Pseudomonas aeruginosa Proteus vulgaris

Staphylococcus aureus Explanation: Staphylococcus aureus causes over 50% of bone infections. Other organisms include Proteus vulgaris and Pseudomonas aeruginosa, as well as E. coli.

A client who has AIDS is being treated in the hospital and admits to having periods of extreme anxiety. What would be the most appropriate nursing intervention?

Teach the client guided imagery.

A day care worker comes to the clinic for mild itching and rash of both hands. The nurse suspects contact dermatitis. The diagnosis is confirmed if the rash appears:

erythematous with raised papules.

Colloid oncotic pressure

osmotic pressure created by the protein (mainly albumin) in the bloodstream (synonym: colloidal osmotic pressure)

After teaching a group of students about the structure of the skin, the nursing instructor determines that the teaching was successful when the group identifies which of the following as the true skin?

dermis

A patient has a moisture-retentive dressing for the treatment of a sacral decubitus ulcer. How long should the nurse leave the dressing in place before replacing it?

12 to 24 hours

The primary nursing intervention that will control swelling while treating a musculoskeletal injury is:

Elevate the affected area. Explanation: Elevation is used to control swelling. It is facilitated by cold, immobilization, and compression. Refer to Box 42-1 in the text.

A nurse at an ophthalmology clinic is providing teaching to a client who has open-angle glaucoma and a new treatment regimen of timolol and pilocarpine eye drops. Which of the following instructions should the nurse provide?

Administer the medications 5 min apart The nurse should instruct the client that, if more than one ophthalmic medication is to be administered, they should be given 5 min apart.

The nurse is caring for a client who may have a lice infestation. The nurse is using a bright light focused on an area of the head to confirm the presence of lice. In which manner is it easiest to differentiate nits from dandruff?

Nits are difficult to move from hair shafts.

Which is useful in identifying acute or chronic tears of the joint capsule or supporting ligaments of the knee, shoulder, ankle, hip, or wrist?

Arthrography Explanation: Arthrography is useful in identifying acute or chronic tears of the joint capsule or supporting ligaments of the knee, shoulder, ankle, hip, or waist. Meniscography is a distractor for this question. Bone densitometry is used to estimate bone mineral density. An EMG provides information about the electrical potential of the muscles and nerves leading to them.

A patient is scheduled for a procedure that will allow the physician to visualize the knee joint in order to diagnose the patient's pain. What procedure will the nurse prepare the patient for?

Arthroscopy Explanation: Arthroscopy is a procedure that allows direct visualization of a joint through the use of a fiberoptic endoscope. Thus, it is a useful adjunct to diagnosing joint disorders.

The nurse is caring for a patient after arthroscopic surgery for a rotator cuff tear. The nurse informs the patient that full activity can usually resume after what period of time?

6 to 12 months The course of rehabilitation following repair of a rotator cuff tear is lengthy (i.e., 6 to 12 months); functionality after rehabilitation depends on the patient's dedication to the rehabilitation regimen (NAON, 2007).

The nurse is conducting a community education program on hip fracture risk. The nurse evaluates that the participants understand the program when the participants determine that client at highest risk for a hip fracture is a(n)

80-year-old man recently widowed. Explanation: Hip fracture occurs with greater incidence in elderly people and is often a life-altering event that has a negative impact on the client's mobility and quality of life.

Which statement describes paresthesia?

Abnormal sensations Explanation: Abnormal sensations, such as burning, tingling, and numbness, are referred to as paresthesias. The absence of muscle tone suggesting nerve damage is referred to as paralysis. A fasciculation is the involuntary twitch of muscle fibers. A muscle that holds no tone is referred to as flaccid.

A client with severe combined immunodeficiency disease is receiving immunosuppression therapy to ensure engraftment of depleted bone marrow during transplantation procedures. What is the priority nursing care for this client?

Meticulous infection control precautions

A patient diagnosed with liver failure has jaundice. Jaundice is often first observed in which of the following areas?

sclerae

The nurse teaches the client who demonstrates herpes zoster (shingles) that

the infection results from reactivation of the chickenpox virus.

osmolality

the number of milliosmoles (the standard unit of osmotic pressure) per kilogram of solvent; expressed as milliosmoles per kilogram (mOsm/kg). (The term osmolality is used more often than osmolarity to evaluate serum and urine.)

Hydrostatic pressure

the pressure created by the weight of fluid against the wall that contains it. In the body, hydrostatic pressure in blood vessels results from the weight of fluid itself and the force resulting from cardiac contraction (synonym: hydraulic pressure)

Which assessment finding would the nurse expect to document for a client with ataxia-telangiectasis?

vascular lesions

A client comes to the dermatology clinic with numerous skin lesions. Inspection reveals that the lesions are elevated, sharply defined, less than 0.5 cm in diameter, and filled with serous fluid. When documenting these findings, the nurse should use which term to describe the client's lesions?

vesicles

The nurse explains to a client that immunotherapy initially starts with injections at which interval?

weekly

The majority of patient with primary immunodeficiency are in which age group?

younger than 20

Based on the nurse's base knowledge of primary immunodeficiencies, how would the nurse complete this statement? Primary immunodeficiencies

develop early in life after protection from maternal antibodies decreases.

A provider prescribes a subcutaneous anabolic agent for an older adult client to prevent fractures associated with osteoporosis. What is the most likely prescribed drug? Alendronic acid Teriparatide Raloxifene Calcitonin

Teriparatide Explanation: Teriparatide (Forteo) is a subcutaneously administered anabolic agent that is taken once daily. The other drug choices are oral preparations.

A client is beginning highly active antiretroviral therapy (HAART). The client demonstrates an understanding of the need for follow up when scheduling a return visit for viral load testing at which time?

6 weeks

Normal calcium concentration

8.8-10.5 mg/dL

Normal chloride concentration

98-106 mEq/L (extracellular) 3-4 mEq/L (intracellular)

Which client is most likely to develop systemic lupus erythematosus (SLE)?

A 27-year-old Black female Explanation: SLE strikes nearly 10 times as many women as men and is most common in women between ages 15 and 40. SLE affects more Black women than white women; its incidence is about 1 in every 250 Black women, compared to 1 in every 700 white women.

While taking the health history of a newly admitted client, the nurse asks for a list of the client's current medications. Which of the following medication classifications would place the client at risk for impaired immune function?

Antimetabolites

A client has breast cancer. The nurse is concerned about a compromised immune system in this client for which reason?

Antineoplastic drug therapy

A client receiving antiretroviral therapy reports "not urinating enough." What is the nurse's best action?

Assess blood urea nitrogen and creatinine.

Which of the following is an appropriate priority nursing diagnosis for the client following an arthrocentesis?

Risk for infection Explanation: The priority nursing diagnosis following an arthrocentesis is risk for infection. The client may experience acute pain. The client needs adequate information before experiencing the procedure. Activity intolerance would not be an expected nursing diagnosis.

A nurse on a surgical unit is caring for four clients who have healing wounds. Which of the following wounds should the nurse expect to heal by primary intention?

Surgical incision With primary intention, a clean wound is closed mechanically, leaving well-approximated edges and minimal scarring. A surgical incision is an example of a wound that heals by primary intention.

A nurse is taking the health history of a newly admitted client. Which of the following conditions would NOT place the client at risk for impaired immune function?

Surgical removal of the appendix

A nurse is caring for a client who has HIV. Which of the following laboratory values is the nurses's priority? Positive Western blot test CD4-T-cell count 180 cells/mm^3 Platelets 150,000/mm^3 WBC 5,000/mm^3

CD4-T-cell count 180 cells/mm^3 A CD4-T-cell count of less than 180 cells/mm^3 indicates that the client is severely immunocompromised and is at high risk for infection. Therefore, this value is the priority for the nurse to report to the provider

The nurse is caring for a client who has had emphysema for 10 years. When performing a fingernail assessment, what does the nurse anticipate the client's nails will be documented as?

Clubbing

Which assessment findings would the nurse expect in the client with osteomalacia?

Column B Explanation: Osteomalacia is characterized by decreased serum calcium and phosphorus and elevated alkaline phosphatase levels.

Which is a neurovascular problem caused by pressure within a muscle area that increases to such an extent that microcirculation diminishes?

Compartment syndrome Explanation: Compartment syndrome is caused by pressure within a muscle area that increases to such an extent that microcirculation diminishes. Remodeling is a process that ensures bone maintenance through simultaneous bone resorption and formation. Hypertrophy is an increase in muscle size. Fasciculation is the involuntary twitch of muscle fibers.

Two days after application of a cast to treat a fractured femur, the client reports severe, deep, and constant pain in the leg. What will the nurse suspect? Chronic venous insufficiency. Infection. Phlebitis. Compartment syndrome.

Compartment syndrome. Explanation: Compartment syndrome refers to the compression of nerves, blood vessels, and muscle within a closed space. This leads to tissue death from lack of oxygenation.

A client received 2 units of packed red blood cells while in the hospital with rectal bleeding. Three days after discharge, the client experienced an allergic response and began to itch and break out with hives. What type of reaction does the nurse understand could be occurring?

Delayed hypersensitivity response

A nurse is admitting a client who has a partial hearing loss. Which of the following is the priority action by the nurse?

Determine if the client uses hearing aids The first action the nurse should take using the nursing process is to assess the client. The nurse should find out if the client has hearing aids and whether they are in place and functioning.

The nurse is completing a physical assessment with an older adult client. Which findings indicate to the nurse that the client is experiencing age-related changes to the immune system? Select all that apply. Crepitus Diarrhea Skin tears Muscle cramps Pain with voiding

Diarrhea Skin tears Pain with voiding

Which diagnostic test would the nurse expect to be ordered for a client with lower extremity muscle weakness?

Electromyograph (EMG) Explanation: The EMG provides information about the electrical potential of the muscles and the nerves leading to them. The test is performed to evaluate muscle weakness, pain, and disability. An arthrocentesis, bone scan, and biopsy does not measure muscle weakness.

tonicity

fluid tension or the effect that osmotic pressure of a solution with impermeable solutes exerts on cell size because of water movement across the cell membrane

The nurse is reviewing various medications with a client that can be used to treat allergic disorders. What medication will the nurse identify as an intranasal corticosteroid?

fluticasone

After completing a skin assessment of an older adult patient, the nurse documents evidence of lentigines, which indicate which of the following?

freckles

Which drug is an oral retinoid used to treat acne?

isotretinoin

While reviewing an older adult's medical record, the nurse notes that the patient has solar lentigo. he nurse interprets this as which of the following?

liver spots

Assessment of a patient reveals a flat and nonpalpable skin lesion that is 0.5 cm with a circumscribed border. The nurse documents this lesion as which of the following?

macule

A 15 year-old pubescent boy is having a sports physical for school. Findings on the face and body indicate that the client is overproducing sebum, which is consistent with the client's age. What is the primary function of sebum?

prevents drying and cracking of the skin and hair

A nurse assesses a client with dry, rough, scaly skin without lesions on the legs. The client reports itching in the affected area. What skin assessment would the nurse document?

pruritus

A patient visits a health clinic with a skin lesion on her right forearm. The lesion is inflamed, swollen, and draining. The nurse practitioner knows the best choice of a treatment would be the application of which of the following?

soothing lotion

Sudoriferous glands secrete which type of substance?

sweat

Which term indicates an accumulation of crystalline depositions in articular surfaces, bones, soft tissue, and cartilage?

Tophi Explanation: Tophi, when problematic, are surgically excised. Subchondral bone refers to a bony plate that supports the articular cartilage. Pannus refers to newly formed synovial tissue infiltrated with inflammatory cells. Joint effusion refers to the escape of fluid from the blood vessels or lymphatic vessels into the joint cavity.

A nurse is teaching a client who is allergic to ragweed. What season does the nurse advise the client to expect an increase in symptoms?

early fall

Which of the following is the first barrier method that can be controlled by the woman?

female condom

A client has had a splenectomy after sustaining serious internal injuries in a motorcycle accident, including a ruptured spleen. Following removal of the spleen, the client will be susceptible to:

infection because the spleen removes bacteria from the blood.

The nurse is working with a client with allergies. What will the nurse use to confirm allergies and decrease the risk of anaphylaxis?

intradermal testing

Which is the principal hardening ingredient of the hair and nails?

keratin

The nurse is caring for a client exposed to peanuts with a known allergy. What assessment is considered the most serious manifestation of angioneurotic edema?

laryngeal swelling

The nurse notes that the client's lower extremities are covered with very dry skin and that the horny layer of the skin has become thickened. The nurse notes the finding as

lichenification

Which response is a humoral immune response?

Anaphylaxis

Development of malignant melanoma is associated with which risk factor?

History of severe sunburn

While interviewing a client with an allergic disorder, the client tells the nurse about an allergy to animal dander. The nurse knows that animal dander is what type of substance?

Complete protein antigen

A client with AIDS is admitted to the hospital with severe diarrhea and dehydration. The physician suspects an infection with Cryptosporidium. What type of specimen should be collected to confirm this diagnosis?

Stool specimen for ova and parasites

The nurse is caring for a patient with an immunodeficiency disorder. What cardinal symptoms of immunodeficiency does the nurse recognize while caring for this patient? (Select all that apply.) Chronic diarrhea Nonproductive cough Chronic or recurrent severe infections Poor response to the treatment with antibiotics Vomiting

Chronic diarrhea Chronic or recurrent severe infections Poor response to the treatment with antibiotics

Kaposi sarcoma (KS) is diagnosed through

biopsy.

Which term refers to an incomplete antigen?

hapten

A client visits the employee health department because of mild itching and a rash on both hands. What will the employee health nurse focus on during the assessment interview?

chemical and latex glove use.

What food can the nurse suggest to the client at risk for osteoporosis?

Broccoli Explanation: Calcium is important for the prevention of osteoporosis. Broccoli is high in calcium.

Nursing students are reviewing information about various types of skin lesions. The students demonstrate understanding of the information when they identify which of the following as a vascular lesion?

spider angioma

A patient was seen in the clinic 3 days previously for allergic rhinitis and was given a prescription for a corticosteroid nasal spray. The patient calls the clinic and tells the nurse that the nasal spray is not working. What is the best response by the nurse?

"The full benefit of the medication may take up to 2 weeks to be achieved."

The nurse is preparing the client for computed tomography. Which information should be given by the nurse?

"You must remain very still during the procedure." Explanation: In computed tomography, a series of detailed x-rays are taken. The client must lie very still during the procedure. A contrast agent, not a radioisotope, may or may not be injected. Arthrocentesis involves the removal of fluid from a joint. A small bit of tissue is removed with a biopsy

A home care nurse is visiting a client with acquired immune deficiency syndrome (AIDS) at home. During the visit, the nurse observes the caregiver providing care. What action by the caregiver would alert the nurse to the need for additional teaching?

Caregiver cleans the client's anal area without wearing gloves

A client with lupus has had antineoplastic drugs prescribed. Why would the physician prescribe antineoplastic drugs for an autoimmune disorder?

For their immunosuppressant effects

The nurse is instructing the parents of a child with head lice. Which statement should the nurse include?

Use shampoo with piperonyl butoxide.

Colloid

a fluid containing particles that are nonsoluble and evenly distributed throughout the solution

A client receiving atazanavir requires what priority intervention?

cardiac assessment

A client is administered foscarnet to treat a case of CMV retinitis. Which adverse effect should the nurse closely monitor in the client?

Electrolyte imbalances

Which statement is accurate regarding isotretinoin?

It is teratogenic in humans

A client reports to a health care provider's office for intradermal allergy testing. Before testing, the nurse provides client teaching. Which client statement indicates a need for further education?

"If I notice tingling in my lips or mouth, gargling may help the symptoms."

A client is concerned about finding a few strands of hair on a pillow after sleeping and additional strands on the brush when styling the hair. Which response will the nurse make regarding the client's concern?

"There are approximately 100 strands of hair lost per day."

A 52-year-old client asks the nurse for interventions for the treatment and prevention of actinic ketatosis. The client is a construction foreman and has actinic ketatosis that is noted only on the right side of the face next to the nose. Which recommendation is appropriate for this client?

Avoidance of direct sunlight with protective clothing measures should be discussed with the client.

Which of the following is a age-related change associated with the immune system?

Decreased antibody production

A nurse is caring for a client who has fractures of the symphysis pubis and pelvis. The nurse should monitor the client for which of the following findings of a common complication of pelvic fractures?

Hematuria Clients who sustain a fracture to the pelvis and symphysis pubis should be monitored for manifestation of internal bleedings, such as blood in the urine and stool

Which adverse effect should the nurse closely monitor in a client who takes immunosuppressive drugs?

Respiratory or urinary system infections

Which diagnostic test is used to examine cells from herpes zoster?

Tzanck smear

Which of the following aggravates the condition caused by acne vulgaris?

cosmetics

The nurse is assessing the skin of a client with tinea pedis and notes a linear crack. The nurse documents this as

fissure

A client with a history of diabetes mellitus has recently developed furunculosis. What is causing the client's condition?

infection

An example of a flat bone is the

sternum. Explanation: An example of a flat bone is the sternum. A short bone is a metacarpal. The femur is a long bone. The vertebra is an irregular bone.

A client with a history of gout experiences an attack every 2 to 3 months despite losing weight and stopping all alcohol intake. Which question will the nurse ask when assessing this client?

"Are you taking the medication as prescribed?" Explanation: Medication adherence is critical but poor among clients prescribed urate lowering therapies for gout. Between acute episodes, the client feels well and may abandon medications and preventive behaviors, which may result in an acute attack. Asking about medication adherence is the appropriate. Exercise, fat-soluble vitamins, and rest periods will not increase the risk of having an attack of gout.

A client diagnosed with carpal tunnel syndrome (CTS) asks the nurse about numbness in the fingers and pain in the wrist. What is the best response by the nurse? "CTS is a neuropathy that is characterized by flexion contracture of the fourth and fifth fingers." "CTS is a neuropathy that is characterized by pannus formation in the shoulder." "CTS is a neuropathy that is characterized by compression of the median nerve at the wrist." "CTS is a neuropathy that is characterized by bursitis and tendinitis."

"CTS is a neuropathy that is characterized by compression of the median nerve at the wrist." Explanation: Carpal tunnel syndrome is an entrapment neuropathy that occurs when the median nerve at the wrist is compressed by a thickened flexor tendon sheath, skeletal encroachment, edema, or a soft tissue mass.

A nurse is providing discharge teaching to a client who is immunosuppressed. Which statement by the client indicates the need for additional teaching?

"I can eat whatever I want as long as it's low in fat."

A nurse is providing preoperative teaching for a client who has colorectal cancer and is to undergo placement of a colostomy with a perineal wound. Which of the following statements by the client indicates an understanding of the teaching?

"I can only have liquids for 2 days before the surgery." The client should consume a full or clear liquid diet for 24 to 48 hours before the surgery to decrease bulk. The client should consume a low-residue diet for several days prior to surgery to decrease peristalsis.

During a routine checkup, a nurse observes the client's skin to be tight and shiny. Which of the following is the correct indication of this sign?

Fluid retention

Which of the following protective responses begin with the B lymphocytes?

Humoral

Which is a major manifestation of Wiskott-Aldrich syndrome?

Thrombocytopenia

The nurse knows the best strategy for latex allergy is

avoidance of latex-based products.

A client has just undergone a leg amputation. What will the nurse closely monitor the client for during the immediate postoperative period?

Hematoma Explanation: Hematoma, hemorrhage, and infection are potential complications in the immediate postoperative period. Sleeplessness, nausea, and vomiting may occur but are adverse reactions, not complications. Chronic osteomyelitis and causalgia are potential complications that are likely to arise in the late postoperative period. A neuroma occurs when the cut ends of the nerves become entangled in the healing scar. This would occur later in the postoperative course.

Which factor causes wrinkles among older adults?

Loss of subcutaneous tissue

A client is receiving ganciclovir as part of the treatment for cytomegalovirus retinitis. What would the nurse monitor the results of the client's laboratory tests for?

neutropenia

A nursing instructor is giving a lecture on the immune system. The instructor's discussion on phagocytosis will include:

neutrophils and monocytes.

The nurse is teaching a client with osteoarthritis about the disease. What is the most important client focus for disease management?

strategies for remaining active Explanation: The goals of osteoarthritis disease management are to decrease pain and stiffness and improve joint mobility. Strategies for remaining active are the most important client focus. The detection of complications, disease-modifying antirheumatic drugs management, and prevention of joint deformity are considerations, but not the most important priorities for the client.

When describing the functions of the skin to a group of nursing students, which skin layer would the instructor include as having the capacity to absorb water?

stratum corneum

The diagnosis of a skin disorder is made chiefly by which of the following?

visual inspection

A client in the emergency department is being treated for a wrist fracture. The client asks why a splint is being applied instead of a cast. What is the best response by the nurse? "It is best if an orthopedic doctor applies the cast." "A splint is applied when more swelling is expected at the site of injury." "Not all fractures require a cast." "You would have to stay here much longer because it takes a cast longer to dry."

"A splint is applied when more swelling is expected at the site of injury." Explanation: Splints are noncircumferential and will not compromise circulation when swelling is expected. A splint is applied to support and immobilize the injured joint. A fracture will swell as part of the inflammation process. The client would not have to stay longer if a fiberglass cast is applied. Fiberglass cast dry in approximately 30 minutes. An orthopedic doctor is not needed to apply the cast. Many nurses and technicians are trained in proper application of a cast. Some fractures may not be treated with a cast but it would not be appropriate to answer with this response because it does not reflect the actual reason for a splint being applied.

During a routine physical examination on an older female client, a nurse notes that the client is 5 feet, 3/8 inches (1.6 m) tall. The client states, "How is that possible? I was always 5 feet and 1/2? (1.7 m) tall." Which statement is the best response by the nurse?

"After menopause, the body's bone density declines, resulting in a gradual loss of height." Explanation: The nurse should tell the client that after menopause, the loss of estrogen leads to a loss in bone density, resulting in a loss of height. This client's history doesn't indicate spinal compression. Telling the client that measuring tools used to obtain the client's height may have a discrepancy or that the posture begins to stoop after middle age doesn't address the client's question.

A child has just been diagnosed with a primary immune deficiency. The parents state, "Oh, no. Our child has AIDS." Which response by the nurse would be most appropriate?

"Although AIDS is an immune deficiency, your child's condition is different from AIDS."

A client comes to the physician's office for treatment of severe sunburn. The nurse takes this opportunity to discuss the importance of protecting the skin from the sun's damaging rays. What is the appropriate teaching by the nurse to prevent skin damage?

"Apply sunscreen even on overcast days."

During an interview, a middle-aged male patient states, "My hair is starting to turn gray." The nurse demonstrates understanding of this change by responding with which statement?

"As you get older, your hair begins to lose its pigment."

When preparing a client with acquired immunodeficiency syndrome (AIDS) for discharge to home, the nurse should be sure to include which instruction?

"Avoid sharing such articles as toothbrushes and razors."

A client is prescribed didanosine as part of a highly active antiretroviral therapy (HAART). Which instruction would the nurse emphasize with this client?

"Be sure to take this drug about 1/2 hour before or 2 hours after you eat."

A client with a recent left above-the-knee amputation states, "I can feel pain in my left toes." Which is the best response by the nurse?

"Describe the pain and rate it on the pain scale." Explanation: The nurse should recognize phantom pain as real and complete a pain assessment as if the limb were attached. The client's pain should be address and treated appropriately. By telling the client that the toes have been amputated or the pain is really from the nerves in the upper leg, the nurse is negating the client's pain. Opioid pain medication can be effective with phantom pain.

The nurse is conducting the admission assessment for a client who is to undergo an arthrogram. What is the priority question the nurse should ask?

"Do you have any allergies?" Explanation: Many contrast dyes contain iodine. Therefore, it is essential for the nurse to determine whether the client has any allergies, especially to iodine, shellfish, and other seafood. Asking about eating or urinating is important but not priority. The claustrophobia is not a concern for the arthrogram.

A client asks why they have a buildup of cerumen despite washing their ears every day. Which statement will the nurse make in response?

"Earwax is made by glands in your ears."

A nurse is providing teaching to a client about preventing skin cancer. Which of the following client statements indicates a need for further teaching? "Eating a high fiber diet will reduce my risk for developing skin cancer." "I should check my skin monthly for any changes." "I should avoid the use of tanning booths." "I should use sunscreen even on cloudy days."

"Eating a high fiber diet will reduce my risk for developing skin cancer." A high-fiber diet is recommended to reduce the risk for colon cancer

A nurse is teaching a client who was recently diagnosed with carpal tunnel syndrome. Which statement should the nurse include?

"Ergonomic changes can be incorporated into your workday to reduce stress on your wrist." Explanation: Ergonomic changes, such as adjusting keyboard height, can help clients with carpal tunnel syndrome avoid hyperextension of the wrist. This condition is associated with repetitive tasks such as clerical work, not sports. The condition may be managed with medications, yoga, acupuncture, and wrist (not arm) splints.

A patient comes to the clinic and asks the nurse why the skin of the forehead, palms, and soles has a yellow-orange tint. There is no yellowing of the sclera or mucous membranes. What should the nurse question the patient regarding?

"Have you been eating a large amount of carotene-rich foods?"

When obtaining a health history from a patient with possible abnormal immune function, what question would be a priority for the nurse to ask?

"Have you ever received a blood transfusion?"

A nurse in a medical clinic is providing teaching to an older adult client who has osteoarthritis that is affecting her knees. Which of the following client statements indicates an understanding of the teaching? "I can use either heat or ice to help relieve the discomfort." "Ibuprofen is the first step in medication therapy for osteoarthritis." "I should limit physical activity to prevent further injury." "I will elevate my legs by placing two pillows under my knees when I go to bed."

"I can use either heat or ice to help relieve the discomfort." The nurse should reinforce that different treatment modalities, such as heat or cold therapy, can be tried to determine which one is more effective for the client. Heat application can help with muscle relaxation in the area around the affected joint. The application of cold numbs nerve endings and decreases joint inflammation.

The nurse is educating a client about allergy management at home. What client statements indicate no further teaching is required? Select all that apply. "I will vacuum my floors once a week." "I bought a wooden chair for my living room." "I picked out a new tufted bedspread for my bed." "I only let my dog sleep with me every other day." "I have pull shades on all of my windows."

"I have pull shades on all of my windows." "I bought a wooden chair for my living room."

A nurse is caring for a client following cataract surgery. Which of the following comments from the client should the nurse report to the client's provider?

"I need something for the pain in my eye. I can't stand it." Following cataract surgery, the client should expect only mild pain and should immediately report any pain, decrease in vision, or increase in discharge from the eye. Severe eye pain after surgery might indicate increased intraocular pressure or hemorrhage.

A nurse is providing teaching to a client who has cancer and is receiving external radiation therapy. Which of the following statements by the client indicates an understanding of the teaching?

"I need to protect the area from sunlight." To prevent skin irritation and subsequent breakdown, the nurse should instruct the client to protect areas of skin from sunlight that receive radiation.

The nurse has educated a patient with low back pain about techniques to relieve the back pain and prevent further complications. What statement by the patient shows understanding of the education the nurse provided? "I will lie prone with my legs slightly elevated." "I will avoid prolonged sitting or walking." "I will bend at the waist when I am lifting objects from the floor." "Instead of turning around to grasp an object, I will twist at the waist."

"I will avoid prolonged sitting or walking." Explanation: The nurse encourages the patient to alternate lying, sitting, and walking activities frequently, and advises the patient to avoid sitting, standing, or walking for long periods.

A nurse is caring for a client who has burn injuries to his trunk. The nurse is explaining what to expect form the prescribed hydrotherapy. Which of the following statements by the client indicates an understanding of the teaching?

"I will be on a special shower table." The special shower table facilitates examination and debridement of the wound during hydrotherapy. An advantage of using the showering technique as opposed to a tub bath is that the water can be kept at a constant temperature and there is a lower risk of wound infection.

The nurse is teaching the client who has an immunodeficiency disorder how to avoid infection at home. Which statement indicates that additional teaching is needed?

"I will be sure to eat lots of fresh fruits and vegetables every day."

A hospice nurse is providing education about palliative care to the partner of a client who has end-stage liver cancer. Which of the following statements by the partner indicates an understanding of teaching?

"I will continue to talk to him even when he's sleeping." The nurse should reinforce to the partner that the client's hearing is thought to be the last sense to leave when in the dying process. Therefore, continue to softly communicate with the client

The nurse instructs a client with polymyositis rheumatica (PMR) about prescribed corticosteroids. Which statement regarding corticosteroid is correct?

"I will take the medication until my health care provider tells me to stop." Explanation: A management concern of clients with PMR is taking the medication as prescribed. Oftentimes clients will take the medication until symptoms improve and then will discontinue the medication. The decision to discontinue the medication should be based on clinical and laboratory findings and the length of the prescription. The dose should not be altered and doses should not be skipped if gastric distress occurs.

A nurse is teaching a client about rheumatoid arthritis. Which statement by the client indicates understanding of the disease process?

"It will get better and worse again." The client demonstrates understanding of rheumatoid arthritis if he expresses that it's an unpredictable disease characterized by periods of exacerbation and remission. There's no cure for rheumatoid arthritis, but symptoms can be managed. Surgery may be indicated in some cases.

An older adult with rheumatoid arthritis limits going out with others because of the need to use a cane. Which response will the nurse make to this client?

"Look at the cane as maintaining your independence." The body image and self-esteem of the older adult with rheumatic disease, combined with underlying depression, may interfere with the use of assistive devices such as canes. The use of adaptive equipment may be viewed by the older adult as evidence of aging rather than as a means of increasing independence. The nurse should focus on the cane as a method to increase independence rather than a sign of approaching old age. Reminding the client of aging are inappropriate responses. Inviting people to visit will not help improve the client's feelings about needing to use a cane for safe ambulation.

A nurse is preparing a client with systemic lupus erythematosus (SLE) for discharge. Which instruction should the nurse include in the teaching plan?

"Monitor your body temperature." Explanation: The nurse should instruct the client to monitor body temperature. Fever can signal an exacerbation and should be reported to the physician. Sunlight and other sources of ultraviolet light may precipitate severe skin reactions and exacerbate the disease. Fatigue can cause a flare-up of SLE. Clients should be encouraged to pace activities and plan rest periods. Corticosteroids must be gradually tapered because they can suppress the function of the adrenal gland. Abruptly stopping corticosteroids can cause adrenal insufficiency, a potentially life-threatening situation.

A client with rheumatoid arthritis arrives at the clinic for a checkup. Which statement by the client refers to the most overt clinical manifestation of rheumatoid arthritis?

"My finger joints are oddly shaped." Explanation: Joint abnormalities are the most obvious manifestations of rheumatoid arthritis. A systemic disease, rheumatoid arthritis attacks all connective tissue. Although muscle weakness may occur from limited use of the joint where the muscle attaches, such weakness isn't the most obvious sign of rheumatoid arthritis; also, it occurs only after joint abnormalities arise. Subcutaneous nodules in the hands, although common in rheumatoid arthritis, are painless. The disease may cause gait disturbances, but these follow joint abnormalities.

A 16-year-old has come to the clinic and asks to talk to a nurse. The teen states that she has become sexually active and is concerned about getting HIV. The teen asks the nurse what she can do to keep from getting HIV. What would be the nurse's best response?

"Other than abstinence, only the consistent and correct use of condoms is effective in preventing HIV."

A nurse is caring for a client who has Meniere's disease and asks if he is allowed to ambulate independently. Which of the following responses should the nurse make?

"Please ring for assistance when you wish to get out of bed" This response is appropriate. With assistance, the client can ambulate safely. Tinnitus, one-sided hearing loss, and vertigo are all manifestations of Ménière's disease that can increase the client's risk of falls when ambulating.

A nurse is providing discharge instructions for a client following cataract surgery with insertion of an intraocular lens. Which of the following instructions should the nurse include?

"Restrict lifting objects greater than 10 pounds." The nurse should instruct the client to restrict lifting objects greater than 10 lb to reduce the risk for increased intraocular pressure.

The nurse is teaching a newly diagnosed client about systemic lupus erythematosus(SLE). What statement by the client indicates the teaching was successful?

"The belief is that it is an autoimmune disorder with an unknown trigger." Systemic lupus erythematosus is believed to be an autoimmune disorder but the triggering mechanism is not known. The disorder is more common in women than in men, most with the disorder in the 3rd or 4th decade of life. The disease is considered the "great imitator" because the clinical signs resemble many other conditions. SLE is a diffuse connective tissue disease that affects multiple body systems.

A nurse is caring for a client who has breast cancer and is receiving a combination of chemotherapy medications. The client expresses confusion about the therapy. Which of the following explanations should the nurse provide?

"The chemotherapy medications act at different stages of cell division so more tumor cells are destroyed." Different chemotherapeutic agents act at various stages of cellular mitosis (division). By combining agents, medication therapy is more effective in stopping or slowing the growth of cancerous cells by interfering with their ability to multiply.

The nurse receives a phone call at the clinic from the family of a client with AIDS. They state that the client started "acting funny" and reported headache, tiredness, and a stiff neck. Checking the temperature resulted in a fever of 103.2°F. What should the nurse inform the family member?

"The client may have cryptococcal meningitis and will need to be evaluated by the health care provider."

A nurse is providing discharge teaching to a client who is postoperative following a right mastectomy for breast cancer. The client will be discharged with two Jackson-Pratt drains. Which of the following information should the nurse include in the teaching?

"The drainage tubes often are removed at the same time as the stitches." The nurse should instruct the client that the provider will remove the drainage tubes at the same time the stitches are removed, usually within 7 to 10 days

A nurse is teaching the parents of an infant about primary immunodeficiencies. Which statement verifies that the parents understand the teaching?

"The majority of primary immunodeficiencies are diagnosed in infancy."

The nurse is assessing a client with a musculoskeletal system condition. Which statement indicates to the nurse that the client is experiencing bone pain?

"The pain feels deep in my legs and keeps me awake at night." Explanation: Bone pain is typically described as a dull, deep ache that is "boring" in nature. This pain is not typically related to movement and may interfere with sleep. Muscular pain is described as soreness or aching and is referred to as "muscle cramps." Joint pain is felt around or in the joint and typically worsens with movement. Fracture pain is sharp and piercing and is relieved by immobilization. Sharp pain may also result from bone infection with muscle spasm or pressure on a sensory nerve.

A client is being prepared for a total hip arthroplasty, and the nurse is providing relevant education. The client is concerned about being on bed rest for several days after the surgery. The nurse should explain what expectation for activity following hip replacement?

"The physical therapist will likely help you get up using a walker the day after your surgery." Explanation: Clients post-THA begin ambulation with the assistance of a walker or crutches within a day after surgery. Wheelchairs are not normally utilized. Baseline levels of mobility are not normally achieved until several weeks after surgery, however.

A client with gout has been advised to lose weight. The client informs the nurse of plans to go on a "guaranteed rapid weight loss" plan that involves fasting and heavy exercise. Which response would be most appropriate?

"There might be some difficulties with your plan and fasting." Explanation: Clients should avoid fasting, low-carbohydrate diets, and rapid weight loss because these measures increase the likelihood of ketone formation, which inhibits uric acid excretion. Gradual weight loss helps reduce serum uric acid levels in clients with gout.

A parent of a child who has been having frequent bouts of tonsillitis brings the child back to the clinic for another sore throat. The parent asks the nurse, "What are tonsils good for anyway?" What is the best response by the nurse?

"These tissues filter bacteria from tissue fluid."

A physician orders an emollient for a client with pruritus of recent onset. The client asks why the emollient should be applied immediately after a bath or shower. How should the nurse respond?

"To prevent evaporation of water from the hydrated epidermis."

A nurse is providing postoperative teaching to a client who is scheduled for cataract surgery. Which of the following information should the nurse include?

"Vision will be greatly improved on the day of surgery." Vision should be greatly improved on the day of surgery. This information should be included in the teaching.

An older adult with rheumatoid arthritis says exercise was not effective. Which response will the nurse make to learn the reason for the failure of this treatment approach?

"What types of exercise were you doing?" Explanation: In an older adult with rheumatoid arthritis, exercise programs may not be instituted or may be ineffective because the client expects results too quickly or fails to appreciate the effectiveness of a program of exercise. Strength training is encouraged in the older adult with chronic diseases. The other questions will not help the nurse understand what type of exercise was used and what it was not effective for the client.

A nurse is caring for a client who has rheumatoid arthritis and tells the nurse that she wears a copper bracelet to help her feel better. Which of the following responses should the nurse make? "Yes, I understand that you feel better wearing your bracelet?" "Why do you think the copper helps with your arthritis?" "Believing objects have powers to make you feel better has no scientific basis" "I think you should rely more on your medication therapy than on your bracelet"

"Yes, I understand that you feel better wearing your bracelet?" The nurse illustrates the therapeutic communication technique of accepting. The nurse demonstrates the knowledge that the bracelet is harmless for the client and shows respect for the client's beliefs.

A nurse is educating a client diagnosed with osteomalacia. Which statement by the nurse is appropriate?

"You may need to be evaluated for an underlying cause, such as renal failure." The client may need to be evaluated for an underlying cause. If an underlying cause is discovered, that will guide the medical treatment. The client needs to maintain an adequate to increased supply of calcium, phosphorus, and vitamin D. Dairy products are a good source of calcium. The client is at risk for pathological fractures and therefore should not engage in vigorous exercise.

A nurse is providing postoperative discharge teaching to a client following a panhysterectomy for uterine cancer. Which of the following information should the nurse include in the teaching?

"You might experience manifestations of menopause." The nurse should inform the client that a panhysterectomy includes the removal of the uterus and the ovaries that might cause manifestations of menopause to occur. Manifestations of menopause include hot flashes, night sweats, and vaginal dryness.

There are major differences between primary and secondary immunodeficiencies. Select the most accurate statement the nurse would use to explain the cause of a secondary immunodeficiency.

"Your immune system was most likely affected by an underlying disease process."

A home health nurse is teaching an older adult client who just had cataract surgery. Which of the following instructions should the nurse include?

"keep your head up and straight" Keeping the head straight and avoiding looking down prevents increasing intraocular pressure.

Health education for a woman over age 50 includes providing information about the importance of adequate amounts of calcium and vitamin D to prevent osteoporosis. Select the daily dosage of calcium and vitamin D that the nurse should recommend. 1,200 mg; 1,000 IU 1,400 mg; 1,200 IU 1,800 mg; 1,600 IU 1,600 mg; 1,400 IU

1,200 mg; 1,000 IU Explanation: The daily recommended dosage is 1,200 mg of calcium and 1,000 IU of vitamin D.

The health care provider has prescribed plicamycin to control serum calcium levels in a client with Paget's disease. The dose prescribed is 25 micrograms per kg. The client weighs 132 lbs. How many milligrams will the nurse expect the client to receive?

1.5 Explanation: The client weighs 60 kg (132 lbs/2.2 lbs per kg). The client will receive 1500 micrograms (60 kg x 25 micrograms/kg). 1500 micrograms/1000 micrograms per mg = 1.5 mg.

The nurse tells the client that if exposure to an allergen occurs around 8:00 AM, then the client should expect a mild or moderate reaction by what time?

10:00 am

The nurse working on a pulmonary unit has an order to administer zafirlukast 20 mg twice a day to a client who was admitted with an exacerbation of asthma. Order: Accolate 20 mg oral (po) two times a day (bid) Dose on hand: 10 mg/tablet How many tablets should the nurse give at each dose?

2

A patient comes to the clinic with pruritus and nasal congestion after eating shrimp for lunch. The nurse is aware that the patient may be having an anaphylactic reaction to the shrimp. These symptoms typically occur within how many hours after exposure?

2 hours

Normal phosphorus concentration

2.5-4.5 mg/dL

Normal bicarbonate concentration

24-31 mEq/L (extracellular) 7-10 mEq/L (intracellular)

When a nurse infuses gamma globulin intravenously, the rate should not exceed

3 mL/min

Normal potassium concentration

3.5-5.0 mEq/L (extracellular) 140-150 mEq/L (intracellular)

Patient education for musculoskeletal conditions for the aging is based on the understanding that there is a gradual loss of bone after a peak of bone mass at age:

30 years. Explanation: Bone mass peaks by about age 30, after which there is a universal and gradual loss of bone.

A client has a rash on the arm that has been treated with an antibiotic without eradicating the rash. What type of examination can be used to determine if the rash is a fungal rash using ultraviolet light?

A Wood's light examination

A client with suspected osteomalacia has a fractured tibia and fibula. What test would give a definitive diagnosis of osteomalacia? Demineralization of the bone A bone biopsy Elevated levels of alkaline phosphatase Increased and decreased areas of bone metabolism

A bone biopsy Explanation: A definitive diagnosis is obtained by bone biopsy. Radiographic studies demonstrate demineralization of the bone. A bone scan detects increased and decreased areas of bone metabolism. Alkaline phosphatase levels are detected from a blood sample.

A client has had a kidney transplant performed for end-stage kidney disease. What type of immune response that T-cell lymphocytes perform is related to this type of surgery?

A cell-mediated response

The nurse is caring for a client with renal failure. Which factor indicates a compromised immune system?

A deficiency of circulating lymphocytes

A clinic nurse is performing a physical assessment on a client who has systemic lupus erythematosus (SLE). Which of the following findings should the nurse expect? A grey colored, non-purpuric papular rash. A dry, red rash across the bridge of the nose and on the cheeks. Pitting edema of the hands and fingers. Subcutaneous nodules on the ulnar side of the arm.

A dry, red rash across the bridge of the nose and on the cheeks. A "butterfly" rash that is dry, red, and raised is characteristic of SLE

A nurse is admitting a client who has multiple myeloma and a white blood cell count of 2,200/mm3. Which of the following foods should the nurse prohibit the family members from bringing to the client?

A fresh fruit basket Raw fruits and vegetables are contraindicated for a client who has neutropenia, as the skin might harbor bacteria that can cause an infection. The nurse should prohibit these foods from entering the client's room.

When performing a skin assessment, the nurse notes a localized skin infection of a single hair follicle. The nurse documents the presence of

A furuncle

A nurse educator is preparing to discuss immunodeficiency disorders with a group of fellow nurses. What would the nurse identify as the most common secondary immunodeficiency disorder?

AIDS

A patient had a total hip replacement. What recommended leg position should the nurse ensure is maintained to prevent prosthesis dislocation? Internal rotation Adduction Abduction Flexion

Abduction Explanation: The nurse educates the patient about protective positioning, which includes maintaining abduction and avoiding internal and external rotation, hyperextension, and acute flexion

A nurse is working in a pediatric clinic. After the nurse gives a hepatitis B immunization to an infant, the parent asks what kind of protection this provides for the child. What is the nurse's best response?

Active acquired immunity, which lasts many years or a lifetime

A nurse assessing a client who has multiple fractures in his left leg notes increasing edema. The nurse should recognize this finding as an early manifestation of which of the following complications?

Acute compartment syndrome Edema is an early manifestation of acute compartment syndrome, which is a complication that involves increased pressure within the fascia that leads to reduced circulation to the affected area.

A nurse is preparing an in-service presentation about primary immunodeficiencies. When describing these conditions, what would the nurse need to integrate into the presentation?

Most cases are typically diagnosed in infancy

A nurse in a provider's office is caring for a client who has a new diagnosis of herpes zoster. The nurse should anticipate a prescription for which of the following medications?

Acyclovir The nurse should anticipate a prescription for acyclovir, an antiviral medication, because it inhibits replication of the virus that causes herpes zoster.

A nurse is caring for a client who has acute osteomyelitis. Which of the following interventions is the nurse's priority?

Administer antibiotics to the client The greatest risk to this client is bacteremia caused by the infection which can lead to septic shock; therefore, the priority intervention is antibiotic therapy. The client might require multiple antibiotics for an extended time

A client is hospitalized for open reduction of a fractured femur. During the postoperative assessment, the nurse notes that the client is restless and observes petechiae on the client's chest. Which nursing action is indicated first? Contact the nursing supervisor. Administer oxygen. Contact the health care provider. Elevate the affected extremity.

Administer oxygen. Explanation: The client is demonstrating clinical manifestations consistent with a fatty embolus. Administering oxygen is the top priority. Elevating the extremity won't alter the client's condition. Notifying the nursing supervisor may be indicated by facility policy after other immediate actions have been taken. The nurse should contact the health care provider after administering oxygen.

The nurse is providing care to a client following a knee arthroscopy. What would the nurse expect to include in the client's plan of care?

Administering the prescribed analgesic. Explanation: After an arthroscopy, the client's entire leg is elevated without flexing the knee. A cold pack is placed over the bulky dressing covering the site where the arthroscope was inserted. A prescribed analgesic is administered as necessary. The client is allowed to resume his or her usual diet as tolerated.

The nurse is educating a patient with allergic rhinitis about how the condition is induced. What should the nurse include in the education on this topic?

Airborne pollens or molds

A nurse is caring for a client with a new diagnosis of Paget's disease. The nurse anticipates the provider will prescribe which of the following medications for this client?

Alendronate Alendronate, a bisphosphonate, decreases bone resorption and minimizes loss of bone density

The nurse is caring for a client with a hip fracture. The physician orders the client to start taking a bisphosphonate. Which medication would the nurse document as given? Teriparatide Alendronate Denosumab Raloxifene

Alendronate Explanation: Alendronate is a bisphosphonate medication. Raloxifene is a selective estrogen receptor modulator. Teriparatide is an anabolic agent, and denosumab is a monoclonal antibody agent.

Which elements of assessment of a traumatic musculoskeletal injury should be included when a client is evaluated. Select all that apply.

All should be included. Explanation: Observing for swelling, external bleeding, or bruising; palpating the peripheral pulses; and checking the sensation of the injured part should all be included.

Which of the following is accurate regarding acquired immunity? Select all that apply. An immunologic response acquired during life but not present at birth Usually develops as a result of exposure to an antigen through immunization Can develop by contracting a disease A nonspecific immunity present at birth Also know as innate immunity

An immunologic response acquired during life but not present at birth Usually develops as a result of exposure to an antigen through immunization Can develop by contracting a disease

Which of the following is the most severe form of hypersensitivity reaction?

Anaphylaxis

The nurse observes diffuse swelling involving the deeper skin layers in a client who has experienced an allergic reaction. The nurse would correctly document this finding as

Angioneurotic edema

A patient was seen in the clinic for hypertension and received a prescription for a new antihypertensive medication. The patient arrived in the emergency department a few hours after taking the medication with severe angioedema. What medication prescribed may be responsible for the reaction?

Angiotensin-Converting Enzyme (ACE) Inhibitor

Which term refers to fixation or immobility of a joint?

Ankylosis Explanation: Ankylosis is the fixation or immobility of a joint. It may result from a disease process or from scarring due to trauma. Hemarthrosis refers to bleeding into a joint. Diarthrodial refers to a joint with two freely moving parts. Arthroplasty refers to replacement of a joint.

Which of the following refers to fixation of a joint?

Ankylosis Explanation: Fixation of a joint, called ankylosis, eliminates friction, but at the drastic cost of immobility. Inflammation is manifested in the joints as synovitis. Pannus has a destructive effect on the adjacent cartilage and bone. Articulations are joints.

When assisting the patient to interpret a negative HIV test result, the nurse informs the patient that the results mean which of the following?

Antibodies to HIV are not present in his blood.

The nurse is instructing the patient in how to apply a corticosteroid cream to lesions on the arm. What intervention can the nurse instruct the patient to do to increase the absorption of the medication?

Apply an occlusive dressing over the site after application.

A client is scheduled for surgery to fuse a joint. The nurse identifies this as which of the following?

Arthrodesis Explanation: An arthrodesis is a surgical procedure to fuse a joint. An osteotomy involves cutting and removing a wedge of the bone to change alignment. An arthroplasty is a total reconstruction or replacement of a joint with an artificial joint. Open reduction internal fixation is accomplished with wire, nails, plate and/or an intramedullary rod to hold bone fragments in place until healing is complete.

Which of the following procedures involves a surgical fusion of the joint?

Arthrodesis Explanation: An arthrodesis is a surgical fusion of the joint. Synovectomy is the excision of the synovial membrane. Tenorrhaphy is the suturing of a tendon. An osteotomy alters the distribution of the weight within the joint.

The nurse is performing a neurological assessment. What will this assessment include?

Ask the client to plantar flex the toes. Explanation: A neurological assessment evaluates sensation and motion. Assessing plantar flexion of the toes would be included in a neurological assessment. Capillary refill, palpation of pulses, and inspecting for edema would be included in a vascular assessment.

The nurse is reviewing medications prescribed for a client with rheumatoid arthritis. Which medications will the nurse expect to be prescribed for this client? Select all that apply.

Aspirin Ibuprofen Methotrexate

What is the priority intervention for a client who has been admitted repeatedly with attacks of gout?

Assess diet and activity at home Clients with gout need to be educated about dietary restrictions in order to prevent repeated attacks. Foods high in purine need to be avoided, and alcohol intake has to be limited. Stressful activities should also be avoided. The nurse should assess to determine what is stimulating the repeated attacks of gout. The other interventions are not appropriate for a client with this problem.

A client comes into the emergency department reporting difficulty walking and loss of muscle control in the arms. Once the nurse begins the physical examination, which assessment should be completed if an immune dysfunction in the neurosensory system is suspected?

Assess for ataxia using the finger-to-nose test and heel-to-shin test

A nurse is providing discharge teaching to a client following open radical prostatectomy. The client is going home with an indwelling urinary catheter. Which of the following statements by the client indicates an understanding of the teaching?

"I will change the catheter drainage bag once each week." The nurse should teach the client how to change the catheter drainage bag and to change the bag at least once each week

A nurse is caring for a client who is in skeletal traction. To prevent the complication of skin breakdown in a client with skeletal traction, what action should be included in the plan of care?

Assess the pin insertion site every 8 hours. Explanation: The pin insertion site should be assessed every 8 hours for inflammation and infection. Loose cover dressings should be applied to pin sites. The client should be encouraged to use the overhead trapeze to shift weight for repositioning. Isometric exercises should be done 10 times an hour while awake.

A nurse is caring for a client following removal of a Morton's neuroma. Which nursing intervention would be most appropriate? Assist the client with incentive spirometry. Assess the surgical dressing. Assist with passive range of motion exercises Perform neurovascular assessment of the hand.

Assess the surgical dressing. Explanation: Morton's neuroma is a foot problem characterized by swelling of the median plantar nerve. The nurse will need to assess the surgical dressing. Assisting with incentive spirometry is not the most important intervention. Range of motion exercises should be active, not passive. A hand assessment is not needed with neuroma removal from the foot.

What intervention is a priority when treating a client with HIV/AIDS?

Assessing fluid and electrolyte balance

Which actions by the nurse demonstrate an understanding of caring for a client in traction? Select all that apply. Assessing the client's alignment in the bed Ensuring that the weights are hanging freely Frequently assessing pain level Removing skeletal traction to turn and reposition the client Placing a trapeze on the bed

Assessing the client's alignment in the bed Ensuring that the weights are hanging freely Frequently assessing pain level Placing a trapeze on the bed

A client asks the nurse what the difference is between osteoarthritis (OA) and rheumatoid arthritis (RA). Which response is correct?

"OA is a noninflammatory joint disease. RA is characterized by inflamed, swollen joints." Explanation: OA is a degenerative arthritis, characterized by the loss of cartilage on the articular surfaces of weight-bearing joints with spur development. RA is characterized by inflammation of synovial membranes and surrounding structures. OA may occur in one hip or knee and not the other, whereas RA commonly affects the same joints bilaterally. RA is more common in women; OA affects both sexes equally.

A nurse on an oncology unit is providing discharge teaching to an adolescent female client who received a bone marrow transplant for leukemia. Which of the following information should the nurse include in the teaching? "Take your temperature twice each day." "You may return to school if you feel strong enough." "It is important to always wear shoes." "Clean your toothbrush weekly with isopropyl alcohol." "Avoid using tampons."

"Take your temperature twice each day." Clients who are postoperative bone marrow transplants are immunocompromised and should continually monitor for manifestations of infection. A temperature that is greater than 38 C (100 F) should be reported immediately to the provider "It is important to always wear shoes." A client who had a bone marrow transplant is immunosuppressed and should wear shoes to prevent injury and decrease the risk for infection "Avoid using tampons." The use of tampons is discouraged because they can disrupt the mucosal layer of the vagina and, if left in too long, can support the growth of bacteria

The nurse identifies a nursing diagnosis of ineffective airway clearance related to pneumocystis pneumonia and increased bronchial secretions for a client with AIDS. Which of the following would be appropriate for the nurse to include in the client's plan of care?

Assist with chest physiotherapy every 2 to 4 hours.

A client with diabetes punctured the foot with a sharp object. Within a week, the client developed osteomyelitis of the foot. The client was admitted for IV antibiotic therapy. How long does the nurse anticipate the client will receive IV antibiotics?

At least 4 weeks Explanation: Identification of the causative organism to initiate appropriate and ongoing antibiotic therapy for infection control. IV antibiotic therapy is administered for at least 4 weeks, followed by another 2 weeks (or more) of IV antibiotics or oral antibiotics.

A nurse is preparing an in-service presentation about human immunodeficiency virus (HIV) for a group of new graduate nurses, including the steps in the process of HIV entering the host cell. What would the nurse describe as the first step?

Attachment

A client is experiencing painful joints and changes in the lungs, heart, and kidneys. For which condition will the nurse schedule this client for diagnostic tests?

Autoimmune disorders Explanation: A hallmark of inflammatory rheumatic diseases is autoimmunity, where the body mistakenly recognizes its own tissue as a foreign antigen. Although focused in the joints, inflammation and autoimmunity also involve other areas. The blood vessels (vasculitis and arteritis), lungs, heart, and kidneys may be affected by the autoimmunity and inflammation. It is unlikely that the client's array of symptoms is being caused by heart disease, vascular diseases, or metabolic disorders.

A client is diagnosed with rheumatoid arthritis. When teaching the client and family about rheumatoid arthritis, the nurse should provide which information?

Autoimmune disorders include connective tissue (collagen) disorders.

Which of the following uses the body's own digestive enzymes to break down necrotic tissues?

Autolytic debridement

An emergency department client is diagnosed with a hip dislocation. The client's family is relieved that the client has not suffered a hip fracture, but the nurse explains that this is still considered to be a medical emergency. What is the rationale for the nurse's statement?

Avascular necrosis may develop at the site if it is not promptly resolved. Explanation: If a dislocation or subluxation is not reduced immediately, avascular necrosis (AVN) may develop. Bone remodeling does not take place because a fracture has not occurred. Realignment does not become more difficult with time and pain would subside with time, not become worse.

The client is about to have a skin test for an allergic disorder. What critical instruction should the nurse give this client?

Avoid antihistamines and cold preparations for 48 to 72 hours before the test.

A nurse is teaching a group of young adult clients about health promotion techniques to reduce the risk of skin cancer. Which of the following instructions should the nurse include?

Avoid exposure to the midday sun. The nurse should instruct the clients to avoid skin exposures to the sun, especially during the midday hours of 1000 to 1600 because sun rays are the strongest at that time.

A nurse is reviewing discharge instructions with a client following a right cataract extraction. Which of the following instructions should the nurse include?

Avoid lifting anything heavier than 4.5 kg (10 lb) for 1 week The nurse should instruct the client to avoid activities that increase intraocular pressure. Therefore, the nurse should instruct the client to avoid lifting anything heavier than 4.5 kg (10 lb) for 1 week following surgery.

A client with Sjogren's syndrome is experiencing xerostomia. Which information will the nurse provide to help alleviate this client's problem? Select all that apply. Avoid smoking Eat small frequent meals Omit spicy and salty foods Limit the intake of alcohol Use antiseptic mouthwash twice a day

Avoid smoking Eat small frequent meals Omit spicy and salty foods Limit the intake of alcohol There is no cure for Sjögren's syndrome, and treatment is aimed at symptom management and improving quality of life. Suggestions to help reduce xerostomia include avoiding smoking, eating small frequent meals, omitting spicy and salty foods, and avoiding excessive alcohol use. Antiseptic mouthwash is not recommended to alleviate the symptom of xerostomia due to its drying effects.

The nurse is conducting discharge teaching for a client who is being discharged from the emergency department after an anaphylactic reaction to peanuts. Which education should the nurse include in the teaching? Select all that apply. Use of sedatives to treat reactions Desensitization to allergen Avoiding allergens Wearing a medical alert bracelet

Avoiding allergens Wearing a medical alert bracelet

A client is to have a hip replacement in 3 months and does not want a blood transfusion from random donors. What option can the nurse discuss with the client?

Bank autologous blood.

During a routine physical examination, a nurse observes a 1-cm (0.4-in) lesion on a client's chest. The lesion is raised and flesh-colored with pearly white borders. The nurse should recognize that this finding is suggestive of which of the following types of skin cancer?

Basal cell carcinoma A basal cell tumor usually begins as a small,waxy nodule with rolled, translucent, pearly borders. Telangiectatic vessels can also be present. As a basal cell tumor grows, it can undergo central ulceration

A nurse is planning an educational program about basal cell carcinoma. Which of the following information should the nurse plan to include?

Basal cell carcinoma has a low incidence of metastasis Basal cell carcinoma is a localized lesion that seldom metastasizes

A client is taking a corticosteroid for the treatment of systemic lupus erythematosus. When the nurse is providing instructions about the medication to the client, what priority information should be included?

Be alert for signs and symptoms of infection and report them immediately to the physician.

A nurse is assigned to care for a client diagnosed with autoimmune or idiopathic thrombocytopenic purpura (ITP). When reviewing the client's plan of care prior to caring for the client, the nurse should recognize that the priority concern in caring for the client is to monitor for... Side effects of immunosuppressants Constipation Fatigue Bleeding

Bleeding Thrombocytopenia refers to a decreased platelet count, which puts the client at risk for bleeding. In ITP, the immune system destroys healthy platelets, thinking they are foreign bodies. Using the airway, breathing, circulation (ABC) priority-setting framework is the priority concern for the nurse when providing care for this client

What treatment option does the nurse anticipate for the patient with severe combined immunodeficiency disease (SCID)?

Bone marrow transplantation

A client with a history of allergic rhinitis comes to the clinic for an evaluation. The client is prescribed triamcinolone. What will the nurse include when teaching the client about this drug?

Be aware that some nasal burning and itching may occur.

Which common problem of the upper extremity results from entrapment of the median nerve at the wrist?

Carpal tunnel syndrome Explanation: Carpal tunnel syndrome is commonly due to repetitive hand activities. A ganglion is a collection of gelatinous material near the tendon sheaths and joints that appears as a round, firm, cystic swelling, usually on the dorsum of the wrist. Dupuytren contracture is a slowly progressive contracture of the palmar fascia. Impingement syndrome is associated with the shoulder and may progress to a rotator cuff tear.

A group of students are reviewing information about cast composition in preparation for a discussion on the advantages and disadvantages of each. The students demonstrate understanding of the topic when they cite which of the following as an advantage of a plaster cast? Longer-lasting Better molding to the client More breathable Quicker drying

Better molding to the client Explanation: Plaster casts require a longer time for drying, but mold better to the client, and are initially used until the swelling subsides. Fiberglass casts dry more quickly, are lighter in weight, longer-lasting, and breathable.

A patient has stepped in a hole in the yard, causing an ankle injury. The ankle is edematous and painful to palpation. How long should the nurse inform the patient that the acute inflammatory stage will last?

Between 24 and 48 hours Explanation: After the acute inflammatory stage (e.g., 24 to 48 hours after injury), intermittent heat application (for 15 to 30 minutes, four times a day) relieves muscle spasm and promotes vasodilation, absorption, and repair.

A nurse is teaching a client who has a new prescription for aspirin to treat rheumatoid arthritis. The nurse should include to monitor for which of the following adverse effects of this medication? Constipation Bleeding Blurred vision Insomnia

Bleeding Aspirin can cause bleeding, tinnitus, gastric ulceration, nausea, and heartburn. The client should monitor and report manifestations of bleeding, such as black tarry stools.

Which condition is associated with impaired immunity relating to the aging client?

Breakdown and thinning of the skin

A client has discussed therapy for his HIV-positive status. What does the nurse understand is the goal of antiretroviral therapy?

Bring the viral load to a virtually undetectable level

A patient diagnosed with Addison's disease would be expected to have which of the following skin pigmentations?

Bronze

Which is usually the most important consideration in the decision to initiate antiretroviral therapy?

CD4+ counts

Which medication directly inhibits osteoclasts, thereby reducing bone loss and increasing bone mass density (BMD)?

Calcitonin Explanation: Calcitonin directly inhibits osteoclasts, thereby reducing bone loss and increased BMD. Raloxifene reduces the risk of osteoporosis by preserving BMD without estrogenic effects on the uterus. Teriparatide has been recently approved by the FDA for the treatment of osteoporosis. Vitamin D increases the absorption of calcium.

A home care nurse assesses for disease complications in a client with bone cancer. Which laboratory value may indicate the presence of a disease complication?

Calcium level of 11.6 mg/dl Explanation: In clients with bone cancer, tumor destruction of bone commonly causes excessive calcium release. When the calcium-excreting capacity of the kidneys and GI tract is exceeded, the serum calcium level rises above normal, leading to hypercalcemia (a serum calcium level greater than 10.2 mg/dl). Hyperkalemia (a potassium level greater than 5 mEq/L) isn't caused by bone cancer and is seldom associated with chemotherapy. Hyponatremia (a sodium level less than 135 mEq/L) and hypomagnesemia (a magnesium level less than 1.3 mg/dl) are potential adverse effects of chemotherapy; these electrolyte disturbances don't result directly from bone cancer.

The nurse is assigned to care for a client who has had an open reduction and internal fixation of a fractured right femur 2 days ago. The nurse is listening to the client's lungs and, when moving the gown, observes petechial hemorrhages on the skin of the chest. What is the first action by the nurse? Increase the intravenous fluids for hemorrhage. Request an antihistamine for the allergic reaction. Call the physician to inform them of the findings. Administer pain medication.

Call the physician to inform them of the findings. Explanation: The findings of the nurse indicate that the client may have a fat embolus, and the physician should be informed immediately. Administration of pain medication is not indicated at this time. The rash is not indicative of an allergic reaction. There is no indication that the rash is related to hemorrhage, and there is no need to increase the IV fluids.

Which of the following is an example of a gliding joint?

Carpal bones in the wrist Explanation: Gliding joints allow for limited movement in all directions and are represented by the joints of the carpal bones in the wrist. Hinge joints permit bending in one direction only and include the knee and elbow. The hip is a ball-and-socket joint. The joint at the base of the thumb is a saddle joint.

A nursing student is preparing a teaching plan for a client with an immunodeficiency disorder. The student is going to include the cardinal symptoms in teaching. Which of the following would the student include? Choose all that apply. Chronic diarrhea Chronic or recurrent severe infections Poor response to treatment of infections Chronic fatigue Facial edema

Chronic diarrhea Chronic or recurrent severe infections Poor response to treatment of infections

A client was climbing a ladder, slipped on a rung, and fell on the right side of the chest. X-ray studies reveal three rib fractures, and the client reports pain with inspiration. What is the anticipated treatment for this client?

Coughing and deep breathing with pillow splinting Explanation: Because these fractures cause pain with respiratory effort, the client tends to decrease respiratory excursions and refrains from coughing. As a result, tracheobronchial secretions are not mobilized, aeration of the lung is diminished, and a predisposition to atelectasis and pneumonia results. To help the client cough and take deep breaths and use an incentive spirometer, the nurse may splint the chest with his or her hands, or may educate the client on using a pillow to temporarily splint the affected site.

The nurse is teaching a client about the characteristics of osteoarthritis. How will the nurse determine the client teaching was successful?

Clients may develop Heberden nodes. Explanation: Heberden nodes are a characteristic finding of osteoarthritis. Swan neck deformity, boutonniere deformity, and ulnar deviation are characteristic of rheumatoid arthritis.

A nurse is teaching a class at a community center to a group of young adult, adult, and older adult clients regarding regular screening recommendations for cancer prevention. Which of the following information should the nurse include? Women should start yearly mammograms at age 30 Clients should have a colonoscopy at age 40 and every 10 years thereafter Clients should have a yearly test for fecal occult blood Women should have a yearly clinical breast examination starting at age 45

Clients should have a yearly test for fecal occult blood According to the American Cancer Society, all clients should have a yearly test to check for fecal occult blood ———————————————————————————— According to the American Cancer Society, women should start yearly mammography at age 40 According to the American Cancer Society, clients should have their first colonoscopy at age 50 and then every 10 years thereafter According to the American Cancer Society, women should start to have a yearly clinical breast examination at age 40

The nurse is caring for a pregnant patient with pregnancy-induced hypertension. When assessing the reflexes in the ankle, the nurse observes rhythmic contractions of the muscle when dorsiflexing the foot. What would the nurse document this finding as?

Clonus Explanation: The nurse may elicit muscle clonus (rhythmic contractions of a muscle) in the ankle or wrist by sudden, forceful, sustained dorsiflexion of the foot or extension of the wrist.

An x-ray demonstrates a fracture in which a bone has splintered into several pieces. Which type of fracture is this? Depressed Compound Impacted Comminuted

Comminuted Explanation: A comminuted fracture may require open reduction and internal fixation. A compound fracture is one in which damage also involves the skin or mucous membranes. A depressed fracture is one in which fragments are driven inward. An impacted fracture is one in which a bone fragment is driven into another bone fragment.

A client is admitted to the emergency room after being hit by a car while riding a bicycle. The client sustained a fracture of the left femur, and the bone is protruding through the skin. What type of fracture does the nurse recognize requires emergency intervention? Oblique Greenstick Compound Spiral

Compound Explanation: A compound fracture is a fracture in which damage also involves the skin or mucous membranes with the risk of infection great. A greenstick fracture is where one side of the bone is broken and the other side is bent; it does not protrude through the skin. An oblique fracture occurs at an angle across the bone but does not protrude through the skin. A spiral fracture twists around the shaft of the bone but does not protrude through the skin.

The nurse is applying a cold towel to a patient's neck to reduce body heat. How does the nurse understand that the heat is reduced?

Conduction

A client is admitted to the hospital with a diagnosis of pneumonia. The client informs the nurse of having several drug allergies. The physician has ordered an antibiotic as well as several other medications for cough and fever. What should the nurse do prior to administering the medications?

Consult drug references to make sure the medicines do not contain substances which the client is hypersensitive.

Which is a circulatory indicator of peripheral neurovascular dysfunction?

Cool skin Explanation: Indicators of peripheral neurovascular dysfunction related to circulation include pale, cyanotic, or mottled skin with a cool temperature. The capillary refill is more than 3 seconds. Weakness and paralysis are related to motion. Paresthesia is related to sensation.

A client has a cast applied to the leg for treatment of a tibia fracture and also has a wound on the leg that requires dressing changes due to drainage. For what should the nurse prepare the client?

Cutting a cast window Explanation: After the cast dries, a cast window, or opening, may be cut. This usually is done when the client reports discomfort under the cast or has a wound that requires a dressing change. The window permits direct inspection of the skin, a means to check the pulse in a casted arm or leg, or a way to change a dressing. A bivalve cast is when the cast is cut in two if the leg swells or if the client is being weaned from a cast, when a sharp x-ray is needed, or as a splint for immobilizing painful joints when a client has arthritis. The cast should not be removed due to the instability of a fracture. The client's condition does not indicate an external fixator is required.

This type of T lymphocyte is responsible for altering the cell membrane and initiating cellular lysis. Choose the T lymphocyte.

Cytotoxic T cell

An experiment is designed to determine specific cell types involved in cell-mediated immune response. The experimenter is interested in finding cells that attack the antigen directly by altering the cell membrane and causing cell lysis. Which cells should be isolated?

Cytotoxic T cells

The nurse is asked to explain to the client the age-related processes that contribute to bone loss and osteoporosis. What is the nurse's best response? Increase in calcitonin Decrease in estrogen Decrease in parathyroid hormone Increase of vitamin D

Decrease in estrogen Explanation: Age related processes that contribute to loss of bone mass and osteoporosis are decreases in estrogen, calcitonin, and vitamin D and an increase in parathyroid hormone.

The homecare nurse is evaluating the musculoskeletal system of a geriatric client whose previous assessment was within normal limits. The nurse initiates a call to the health care provider and/or emergency services when which change is found?

Decreased right-sided muscle strength Explanation: Although symmetrical decreases in muscle strength can be a part of the aging process, asymmetrical decreases are not. The nurse should contact the health care provider when decreased right-sided muscle strength is found, as this could indicate a stroke or transient ischemic attack. Decreased flexibility, decreased agility, and increased joint stiffness are all part of the aging process and therefore do not require the nurse to contact the health care provider.

The nurse is planning teaching for a client with gout. Which topics will the nurse include in the teaching? Select all that apply. Decreasing alcohol intake Weight loss Avoiding purine-rich foods Limiting exercise Restricting the intake of water

Decreasing alcohol intake Weight loss Avoiding purine-rich foods Management between the attacks of gout include lifestyle changes to include weight loss, decreasing alcohol intake, and avoiding purine-rich foods. Exercise does not need to be limited and water does not need to be restricted.

A client comes to the emergency department complaining of pain in the right leg. When obtaining the history, the nurse learns that the client has a history of obesity and hypertension. Based on this information the nurse anticipates the client having which musculoskeletal disorder? Muscular dystrophy Paget's disease Degenerative joint disease Scoliosis

Degenerative joint disease Explanation: Obesity predisposes the client to degenerative joint disease. Obesity isn't a predisposing factor for muscular dystrophy, scoliosis, or Paget's disease.

A client with common variable immunodeficiency (CVID) comes to the ED reporting tingling and numbness in the hands and feet, muscle weakness, fatigue, and chronic diarrhea. An assessment reveals abdominal tenderness, weight loss, and loss of reflexes. A gastric biopsy shows lymphoid hyperplasia of the small intestine and spleen as well as gastric atrophy. Based on these findings, what common secondary problem has this client developed?

Pernicious anemia

A client is admitted with cellulitis and experiences a consequent increase in white blood cell count. During what process will pathogens be engulfed by white blood cells that ingest foreign particles?

Phagocytosis

The nurse is assessing a client's skin when the client points out a mole. The nurse brings the mole to the physician's attention when which characteristic is noted?

Diameter exceeding 6 mm

The nurse in an orthopedic clinic is caring for a new client. What sign or symptom would lead a nurse to suspect that a client has a rotator cuff tear? Ability to stretch arm over the head Minimal pain with movement Difficulty lying on affected side Pain worse in the morning

Difficulty lying on affected side Explanation: Clients with a rotator cuff tear experience pain with movement and limited mobility of the shoulder and arm. They especially have difficulty with activities that involve stretching their arm above their head. Many clients find that the pain is worse at night and that they are unable to sleep on the affected side.

A client who is being treated for complications related to acquired immunodeficiency disorder syndrome (AIDS) is receiving interferon parenterally as adjunctive therapy. Why does the nurse understand this route is being used?

Digestive enzymes destroy its protein structure.

There are many ethical issues in the care of clients with HIV or HIV/AIDS. What is an ethical issue healthcare providers deal with when caring for clients with HIV/AIDS?

Disclosure of the client's condition

Which of the following disorders results in widespread hemorrhage and microthrombosis with ischemia?

Disseminated intravascular coagulation (DIC) Explanation: DIC is a systemic disorder that results in widespread hemorrhage and microthrombosis with ischemia. AVN of the bone occurs when the bone loses its blood supply and dies. CRPS is a painful sympathetic nervous system problem. FES occurs when the fat globules released when the bone is fractured occludes the small blood vessels that supply the lungs, brain, kidneys, and other organs.

Which of the following disorders results in widespread hemorrhage and microthrombosis with ischemia? Disseminated intravascular coagulation (DIC) Complex regional pain syndrome (CRPS) Avascular necrosis (AVN) Fat embolism syndrome (FES)

Disseminated intravascular coagulation (DIC) Explanation: DIC is a systemic disorder that results in widespread hemorrhage and microthrombosis with ischemia. AVN of the bone occurs when the bone loses its blood supply and dies. CRPS is a painful sympathetic nervous system problem. FES occurs when the fat globules released when the bone is fractured occludes the small blood vessels that supply the lungs, brain, kidneys, and other organs.

A client who is HIV positive is receiving highly active antiretroviral therapy (HAART) that includes a protease inhibitor (PI). The client comes to the clinic for a follow-up visit. Assessment reveals lipoatrophy of the face and arms. The client states, "I'm thinking the side effects of the drug are worse than the disease. Look what's happening to me." The nurse would most likely identify which nursing diagnosis as the priority?

Disturbed body image related to loss of fat in the face and arms

The nurse caring for a client, who has been treated for a hip fracture, instructs the client not to cross their legs and to have someone assist with tying their shoes. Which additional instruction should the nurse provide to client? Do not flex the hip more than 30 degrees. Do not flex the hip more than 120 degrees. Do not flex the hip more than 90 degrees. Do not flex the hip more than 60 degrees.

Do not flex the hip more than 90 degrees. Explanation: Proper alignment and supported abduction are encouraged for hip repairs. Flexion of the hip more than 90 degrees can cause damage to the a repaired hip fracture.

When assessing the skin of a client with allergic contact dermatitis, the nurse would most likely expect to find irritation at which area?

Dorsal aspect of the hand

The nurse is educating a group of students about peroneal nerve damage. The nurse knows that which assessment will show this type of nerve damage?

Dorsiflexion of the foot and extension of the toes Explanation: Assessment of peripheral nerve function has two key elements: evaluation of sensation and evaluation of motion. To assess motion of the peroneal nerve, the client should be asked to dorsiflex the foot and extend the toes. Pricking the skin along the top of the index finger assesses sensation of the median nerve. Having the client stretch the thumb away from the wrist assesses motion of the radial nerve. Pricking the skin between the medial and lateral surface of the sole will assess tibial nerve sensation.

A client comes to the clinic complaining of low back pain radiating down the left leg. After diagnostic studies rule out any pathology, the health care provider orders a serotonin-norepinephrine reuptake inhibitor (SNRI). Which medication does the nurse anticipate educating the client about? Cyclobenzaprine Gabapentin Amitriptyline Duloxetine

Duloxetine Explanation: Nonprescription analgesics such as acetaminophen (Tylenol) and nonsteroidal anti-inflammatory drugs (NSAIDs) and short-term prescription muscle relaxants (e.g., cyclobenzaprine [Flexeril]) are effective in relieving acute low back pain. Tricyclic antidepressants (e.g., amitriptyline [Elavil) and the newer dual-action serotonin-norepinephrine reuptake inhibitors (e.g., duloxetine [Cymbalta]) (Karp et al., 2010) or atypical seizure medications (e.g., gabapentin [Neurontin], which is prescribed for pain from radiculopathy) are used effectively in chronic low back pain.

Which term refers to a flexion deformity caused by a slowly progressive contracture of the palmar fascia?

Dupuytren's contracture Explanation: Dupuytren's disease results in a slowly progressive contracture of the palmar fascia, called Dupuytren's contracture. A callus is a discretely thickened area of skin that has been exposed to persistent pressure or friction. A hammertoe is a flexion deformity of the interphangeal joint, which may involve several toes. Hallux valgus is a deformity in which the great toe deviates laterally.

The nurse is performing a neurovascular assessment of a client's injured extremity. Which would the nurse report?

Dusky or mottled skin color Explanation: Normally, skin color would be similar to the color in other body areas. Pale or dusky skin color indicates an abnormality that needs to be reported. Presence of pulses, capillary refill of 3 seconds, and warm skin are normal findings.

The nurse is reviewing data collected during the assessment of a client. Which finding about the client's skin condition is genetically based?

Eczema

A client seeks medical attention for the development of a rash on the hands. Which assessment findings indicate to the nurse that the client is experiencing irritant contact dermatitis? Select all that apply. Blisters Edema Burning Redness Crust formation

Edema Burning Redness

A public health nurse is preparing an educational campaign to address a recent local increase in the incidence of HIV infection. The nurse should prioritize what intervention?

Educational programs that focus on control and prevention

A nurse is developing a plan of care for a client who has cellulitis of the leg. Which of the following interventions should the nurse include in the plan?

Elevate the affected leg on two pillows Cellulitis is an acute inflammation of the deep connective tissue of the skin, caused by infection. The edema of the inflammatory response puts the client at risk for skin breakdown. Elevation of the affected area and frequent repositioning reduces depended edema and assist in the healing process

The nurse is reviewing the diagnostic test findings of a client with rheumatoid arthritis. What would the nurse expect to find?

Elevated erythrocyte sedimentation rate The erythrocyte sedimentation rate (ESR) may be elevated, particularly as the disease progresses. ESR shows inflammation associated with RA. Red blood cell count and C4 complement component are decreased. Serum protein electrophoresis may disclose increased levels of gamma and alpha globulin but decreased albumin.

Which is an appropriate nursing intervention in the care of the client with osteoarthritis?

Encourage weight loss and an increase in aerobic activity Weight loss and an increase in aerobic activity such as walking, with special attention to quadriceps strengthening, are important approaches to pain management. Clients should be assisted to plan their daily exercise at a time when the pain is least severe, or plan to use an analgesic, if appropriate, before an exercise session. Gastrointestinal complications, especially bleeding, are associated with the use of nonsteroidal anti-inflammatory drugs. Topical analgesics such as capsaicin and methyl salicylate may be used for pain management.

A nurse in an oncology clinic is assessing a client who has early stage Hodgkin's lymphoma. Which of the following findings should the nurse expect?

Enlarged lymph nodes Hodgkin's lymphoma is a malignancy of lymphoid tissue found in the lymph nodes, spleen, liver, and bone marrow. The first manifestation of this cancer is often an enlarged painless lymph node, or nodes, which appear without a known cause. Other early manifestations include night sweats, unexplained weight loss, fever, and pruritus. The disease can spread to adjacent lymph nodes and later might spread outside the lymph nodes to the lungs, liver, bones, or bone marrow. The spread of Hodgkin's lymphoma is usually in an ordered pattern.

A client with a right below-the-knee amputation is being transferred from the postanesthesia care unit to a medical-surgical unit. What is the highest priority nursing intervention by the receiving nurse? Review the physician's orders for type and frequency of pain medication. Delegate the gathering of enough pillows for proper positioning and comfort. Document the receiving report from the transferring nurse. Ensure that a large tourniquet is in the room.

Ensure that a large tourniquet is in the room. Explanation: The client with an amputation is at risk for hemorrhage. A tourniquet should be placed in plain sight for use if the client hemorrhages. Documenting the receiving report is important but is not the highest priority. The nurse may delegate to unlicensed assistive personnel (UAP) the job of gathering more pillows for positioning, but this is not the highest priority. The nurse will need to review the physician's orders for pain medication, but again, this is not the highest priority, because any hemorrhaging by the client needs to be addressed first.

A nurse is monitoring a client who has cancer and is receiving chemotherapy by peripheral IV infusion. The client reports pain at the insertion site and the nurse notes fluid leaking around the catheter. Which of the following actions should the nurse take first?

Stop the infusion The nurse should apply the urgent versus nonurgent priority-setting framework. Using this framework, the nurse should consider urgent needs the priority need because they pose more of a threat to the client. The nurse might also need to use Maslow's hierarchy of needs, the ABC priority-setting framework, or nursing knowledge to identify which finding is the most urgent. Many chemotherapy medications are vesicants that can cause extensive tissue damage if extravasation occurs; therefore, the nurse's first action should be to stop the infusion immediately.

The nurse is planning an education program for women of childbearing years. What does the nurse recognize as the primary prevention of osteoporosis? Ensuring adequate calcium and vitamin D intake Engaging in non-weight-bearing exercises daily Undergoing assessment of serum calcium levels every year Having a DXA beginning at age 35 years

Ensuring adequate calcium and vitamin D intake Explanation: Nutritional intake of calcium and vitamin D are essential for the prevention of osteoporosis.

A nurse is talking with a client whose provider just told her that she has osteosarcoma. Which of the following nonverbal expressions warrants further investigation by a nurse?

Enthusiasm It is unlikely that a client would appear enthusiastic after receiving a grim diagnosis. The nurse should determine the client's understanding of the diagnosis and initiate any appropriate referrals.

Which blood test confirms the presence of antibodies to HIV?

Enzyme immunoassay (EIA)

A client is suspected of having an immune system disorder. The health care provider wants to perform a diagnostic test to confirm the diagnosis. What test should the nurse prepare the client for?

Enzyme-linked immunosorbent assay

A client suspected of having human immunodeficiency virus (HIV) has blood drawn for a screening test. What is the first test generally run to see if a client is, indeed, HIV positive?

Enzyme-linked immunosorbent assay (ELISA)

Six weeks after an above-the-knee amputation (AKA), a client returns to the outpatient office for a routine postoperative checkup. During the nurse's assessment, the client reports symptoms of phantom pain. What should the nurse tell the client to do to reduce the discomfort of the phantom pain?

Take opioid analgesics as prescribed. Explanation: Opioid analgesics may be effective in relieving phantom pain. Heat, immobility, and elevation are not noted to relieve this form of pain

A client's antibody test for HIV showed no antibodies. For which reason would the client need to have a nucleic acid test completed before being told that testing for HIV is negative?

The client may be in stage 0 of the disease

The emergency room nurse is reporting the location of a fracture to the client's primary care physician. When stating the location of the fracture on the long shaft of the femur, the nurse would be most correct to state which terminology locating the fractured site?

The fracture is on the diaphysis. Explanation: A fracture that is on the diaphysis is understood to be chiefly found in the long shafts of the arms and legs. The epiphyses are rounded, irregular ends of the bones. Saying a fracture is ventrally located does not assist in providing adequate details of the location of the fracture. A tuberosity is a projection from the bone or a protuberance.

The nurse is caring for a female patient who has an exacerbation of lupus erythematosus. What does the nurse understand is the reason that females tend to develop autoimmune disorders more frequently than men?

Estrogen tends to enhance immunity.

A nurse would identify that a colleague needs additional instruction on standard precautions when the colleague exhibits what behavior?

The nurse puts on a second pair of gloves over soiled gloves while performing a bloody procedure.

Which group is at the greatest risk for osteoporosis? European American women Asian American women Men African American women

European American women Explanation: Small-framed, nonobese European American women are at greatest risk for osteoporosis. Asian American women of slight build are at risk for low peak bone mineral density. African American women, who have a greater bone mass than European American women and Asian American Women, are less susceptible to osteoporosis. Men have a greater peak bone mass and do not experience sudden estrogen reduction.

Which statement accurately reflects current stem cell research?

The stem cell is known as a precursor cell that continually replenishes the body's entire supply of both red and white cells.

A dermatologist recommends an over-the-counter suspension to relieve pruritus. The nurse advises the patient that the lotion should be applied:

Every 3 to 4 hours for sustained effectiveness.

Which statement describes the clinical manifestations of a delayed hypersensitivity (type IV) allergic reaction to latex?

They are localized to the area of exposure, usually the back of the hands.

A nurse is teaching a female client who has a new diagnosis of systemic lupus erythematosus (SLE). The nurse should recognize the need for further teaching when the client identifies which of the following as a factor that can exacerbate SLE? Sunlight Pregnancy Infection Exercise

Exercise Deconditioning and muscle atrophy occurs as a result of lack of mobility. The nurse should encourage the client to engage in conditioning exercises alternated with periods of rest

A nurse is taking health history from a new client, which includes asking about a history of blood transfusions. This is important for which reason?

Exposure to foreign antigens may cause altered immune function.

A nurse is caring for a client who has lung cancer that has metastasized. Which of the following findings indicates the client is developing superior vena cava syndrome?

Facial edema Superior vena cava syndrome is a medical emergency resulting from a partial occlusion of the superior vena cava, leading to a decreased blood flow through the vein. Most cases of superior vena cava syndrome are associated with cancers involving the client's upper chest (e.g., advanced lung and breast cancers and lymphoma). The earliest manifestations of superior vena cava syndrome are facial and upper extremity edema. Death can result if the compression is not corrected.

A nurse assesses a client in the health care provider's office. Which assessment findings support a suspicion of systemic lupus erythematosus (SLE)?

Facial erythema, pericarditis, pleuritis, fever, and weight loss Explanation: An autoimmune disorder characterized by chronic inflammation of the connective tissues, SLE causes fever, weight loss, malaise, fatigue, skin rashes, and polyarthralgia. Nearly half of clients with SLE have facial erythema, (the classic butterfly rash). SLE also may cause profuse proteinuria (excretion of more than 0.5 g/day of protein), pleuritis, pericarditis, photosensitivity, and painless mucous membrane ulcers. Weight gain, hypervigilance, hypothermia, and edema of the legs and arms don't suggest SLE.

The femur fracture that commonly leads to avascular necrosis or nonunion because of an abundant supply of blood vessels in the area is a fracture of the:

Femoral neck. Explanation: A fracture of the neck of the femur may damage the vascular system and the bone will become ischemic. Therefore, a vascular necrosis is common.

A client is recovering from a fractured hip. What would the nurse suggest that the client increase intake of to facilitate calcium absorption from food and supplements?

Vitamin D Explanation: The nurse must advise a client recovering from a fractured hip to increase the intake of vitamin D, because vitamin D protects against bone loss and decreases the risk of recurring fracture by facilitating calcium absorption from food and supplements. Amino acids and vitamin B6, though important, do not facilitate the absorption of calcium. Dairy products also do not facilitate the absorption of calcium; however, the exception to this is vitamin D-fortified milk.

While assessing a patient at the clinic the nurse notes patchy, milky white spots. The nurse knows that this finding is a symptom of what?

Vitiligo

T-cells can be either regulator T cells or effector T cells. Regulator T cells are made up of helper and suppressor cells. What function are helper T-cells important in?

Fighting infection

A school nurse is assessing a child for pediculosis capitis. Which of the following manifestations should the nurse recognize as an indication of this condition?

Firmly attached white particles on the hair Pediculus capitis, or head lice, are tiny parasitic insects that live on the scalp and can be spread by close contact with other people. Their eggs (nits) appear much like flakes of dandruff but stick firmly to the hair shaft instead of flaking off of the scalp

The nurse is reading the physician's report of an elderly client's physical examination. The client demonstrates xanthelasma, which refers to which symptom?

Yellowish waxy deposits on the eyelids

The nurse is performing a musculoskeletal assessment for a client whose right leg muscles exhibit no tone and are limp. Which descriptor should the nurse use to document this condition?

Flaccid Explanation: The term flaccid describes muscles that have no tone or are limp. Spastic describes muscles that have greater-than-normal tone. Atonic describes muscles that are not enervated and become soft and flabby. Atrophic describes muscles deterioration that occurs with lack of use and exercise.

A patient has had a stroke and is unable to move the right upper and lower extremity. During assessment the nurse picks up the arm and it is limp and without tone. How would the nurse document this finding?

Flaccidity Explanation: A muscle that is limp and without tone is described as flaccid; a muscle with greater-than-normal tone is described as spastic. Conditions characterized by lower motor neuron destruction (e.g., muscular dystrophy), denervated muscle becomes atonic (soft and flabby) and atrophies.

The nurse is preparing to perform a musculoskeletal assessment for a client with chronic muscle pain. Which assessment technique would be an appropriate tool to evaluate this type of pain?

Flex the bicep against resistance. Explanation: Evaluating muscle strength is a part of the musculoskeletal system. Strength of the bicep muscles can be tested by having the client flex the bicep against resistance. Palpating for the balloon sign assesses for fluid around the knee joint. Measuring the girth of the thigh evaluates for muscle size. Cracking with movement may indicate a ligament slipping over a bony prominence.

Dupuytren's contracture causes flexion of which area(s)? Index and middle fingers Thumb Ring finger Fourth and fifth fingers

Fourth and fifth fingers Explanation: Dupuytren's contracture causes flexion of the fourth and fifth fingers, and frequently the middle finger.

The nurse is preparing a teaching plan for a client with an immunodeficiency. What aspect would the nurse emphasize as most important?

Frequent and thorough handwashing

The nurse is developing a plan of care for a client with toxic epidermal necrolysis (TEN) or Stevens-Johnson syndrome. Which action should the nurse include?

Frequently inspect the oral cavity.

A nurse is planning care for a client who is being treated with chemotherapy and radiation for metastatic breast cancer, and who has neutropenia. The nurse should include which of the following restrictions in the client's plan of care? All visitors from entering the client's room Fresh flowers and potted plants in the room Oral fluid intake to between meals only Activities that could result in bleeding

Fresh flowers and potted plants in the room Clients who are receiving chemotherapy and radiation therapy are likely to become immunocompromised as a result of neutropenia, a decreased white blood cell (WBC) count. Because micro-organisms are likely to be present on fresh flowers and plants, immunocompromised clients are instructed not to accept such gifts into the room. In addition, the client is instructed to eat only thoroughly cooked meats and thoroughly washed fruits and vegetables. Immunocompromised clients are more susceptible to infection and illness from food-borne bacteria than other clients

A client is brought in by ambulance to the emergency department after being involved in a motorcycle accident. The client has an open fracture on his tibia. The wound is highly contaminated and there is extensive soft- tissue damage. How would this client's fracture likely be graded? Grade III Grade IV Grade II Grade I

Grade III Explanation: Open fractures are graded according to the following criteria. Grade I is a clean wound less than 1 cm long. Grade II is a larger wound without extensive soft-tissue damage. Grade III is highly contaminated, has ext

A nurse is caring for a client who is to start taking cyclosporine following a kidney transplant. The nurse should instruct the client that which of the following foods can have an adverse interaction with this medication? Pepperoni Orange juice Grapefruit juice Smoked salmon

Grapefruit juice Clients taking cyclosporine should avoid drinking grapefruit juice because it can increase the therapeutic effect leading to renal and hepatic toxicity

A client is prescribed an oral corticosteroid for 2 weeks to relieve asthma symptoms. The nurse educates the client about side effects, which include

adrenal suppression

A client is prescribed antihistamines, and asks the nurse about administration and adverse effects. The nurse should advise the client to avoid:

alcohol

When learning about HIV/AIDS, the student should be able to differentiate the two subtypes of virus by which characteristic?

HIV-1 is more prevalent than HIV-2 subtypes

In which deformity does the great toe deviate laterally? Pes cavus Hallux valgus Hammertoe Plantar fasciitis

Hallux valgus Explanation: Hallux valgus is a deformity in which the great toe deviates laterally. A hammertoe is a flexion deformity of the interphalangeal joint, which may involve several toes. Pes cavus refers to a foot with an abnormally high arch and a fixed equines deformity of the forefoot. Plantar fasciitis is an inflammation of the foot-supporting fascia.

During a routine physical examination of a client, the nurse observes a flexion deformity of the proximal interphalangeal (PIP) joint of two toes on the right foot. How would the nurse document this finding? Mallet toe Hallux valgus Bunion Hammer toe

Hammer toe Explanation: Hammer toe is a flexion deformity of the proximal interphalangeal (PIP) joint and may involve several toes. Mallet toe is a flexion deformity of the distal interphalangeal joint (DIP), and also can affect several toes. Hallux valgus, also called a bunion, is a deformity of the great (large) toe at its metatarsophalangeal joint.

A client who has been diagnosed with osteoarthritis asks if he or she will eventually begin to notice deformities in the hands and fingers as the condition progresses. Which concept should the nurse include in the response?

Hand and finger deformities are associated with the development of rheumatoid arthritis. Explanation: The nurse should explain to the client that joint deformities occur with rheumatoid arthritis, not osteoarthritis. Osteoarthritis typically follows a pattern of cartilage destruction and increased pain. The nurse is part of the interdisciplinary health care team and is capable of answering the client's questions about the typical progression of disease.

The client with osteoarthritis is seen in the clinic. Which assessment finding indicates the client is having difficulty implementing self-care? Shows increased joint flexibility Has a weight gain of 5 pounds Reports decreased joint pain Reports ability to perform ADLs

Has a weight gain of 5 pounds Explanation: Obesity is a risk factor for osteoarthritis. Excess weight is a stressor on the weight-bearing joints. Weight reduction is often a part of the therapeutic regimen.

A healthcare provider asks a nurse to test a client for Tinel's sign to diagnose carpal tunnel syndrome. What should the nurse do to perform this assessment?

Have the client hold the palm of the hand up while the nurse percusses over the median nerve. Explanation: If tingling, numbness, or pain is felt when the median nerve is percussed, then Tinel's sign is considered positive. To test for Tinel's sign have the client hold the palm of the hand up while the nurse percusses over the median nerve. The client making a fist and pushing will test strength resistance. The client stretching fingers around a ball will not test for Tinel's sign. Having the client pronate the hand and palpating the radial nerve is not Tinel's sign used for carpal tunnel syndrome diagnosis.

A client has been prescribed alendronate for the prevention of osteoporosis. Which is the highest priority nursing intervention associated with the administration of the medication?

Have the client sit upright for at least 30 minutes following administration Explanation: While all interventions are appropriate, the highest priority is having the client sit upright for 60 minutes following administration of the medication. This will prevent irritation and potential ulceration of the esophagus. The client should have adequate intake of vitamin D and obtain yearly dental exams. The concurrent use of corticosteroids and alendronate is link to a complication of osteonecrosis.

A patient in pelvic traction needs circulatory status assessed. How should the nurse assess for a positive Homans' sign?

Have the patient extend each leg and dorsiflex each foot to determine if pain or tenderness is present in the lower leg. Explanation: The nurse should assess for pain on passive flexion of each foot, which could indicate deep vein thrombosis.

Which condition is an early manifestation of HIV encephalopathy?

Headache

Which of the following indicates that a client with HIV has developed AIDS?

Herpes simplex ulcer persisting for 2 months

The nurse is assisting with the collection of a Tzanck smear. What is the suspected diagnosis of the patient?

Herpes zoster

Which factor inhibits fracture healing? History of diabetes Increased vitamin D and calcium in the diet Immobilization of the fracture Age of 35 years

History of diabetes Explanation: Factors that inhibit fracture healing include diabetes, smoking, local malignancy, bone loss, extensive local trauma, age greater than 40, and infection. Factors that enhance fracture healing include proper nutrition, vitamin D and calcium, exercise, maximum bone fragment contact, proper alignment, and immobilization of the fracture.

A nurse is preparing to administer ophthalmic solution to a client. Which of the following actions should the nurse take?

Hold the ophthalmic solution 2 cm (3/4 in) above the lower conjunctival sac The nurse should hold the bottle of ophthalmic solution 1 to 2 cm (1/2 to 3/4 in) above the lower conjunctival sac.

Which sedative medication is effective for treating pruritus?

Hydroxyzine

Which assessment suggests to the nurse that a client with systemic lupus erythematous is having renal involvement?

Hypertension Hypertension is suggestive of renal damage in the client with systemic lupus erythematous.

A patient is hospitalized with a severe case of gout. The patient has gross swelling of the large toe and rates pain a 10 out of 10. With a diagnosis of gout, what should the laboratory results reveal?

Hyperuricemia Explanation: Gout is caused by hyperuricemia (increased serum uric acid).

A client who is HIV positive is experiencing severe diarrhea. Which laboratory test result would the nurse expect to find?

Hypokalemia

The nurse is monitoring a patient who sustained a fracture of the left hip. The nurse should be aware that which kind of shock can be a complication of this type of injury? Hypovolemic Neurogenic Septic Cardiogenic

Hypovolemic Explanation: In a client with a pelvic fracture, the nurse should be aware of the potential for hypovolemic shock resulting from hemorrhage. Cardiogenic shock, in which the heart cannot pump enough blood to meet the body's needs, often arises from severe myocardial infarction. Neurogenic shock is often a consequence of spinal cord injury and resulting loss of sympathetic nervous system function. Septic shock results from body-wide infection.

A patient has suffered a femoral shaft fracture in an industrial accident. What is an immediate nursing concern for this patient?

Hypovolemic shock Explanation: Frequently, the patient develops shock, because the loss of 1,000 mL of blood into the tissues is common with fractures of the femoral shaft (ENA, 2013).

A nurse is assessing a client who is bedridden and was admitted from home. The nurse notes a shallow crater in the epidermis of the client's sacral area. The nurse should document that the client has a pressure ulcer at which of the following stages?

II With a stage II pressure ulcer, there is partial-thickness skin loss involving the epidermis and the dermis. The ulcer is visible and superficial and can appear as an abrasion, blister, or shallow crater. Edema persists, and the ulcer might become infected. The client might report pain, and there might be a small amount of drainage.

A client with ataxia-telangiectasia is admitted to the unit. The nurse caring for the client would expect to see what included in the treatment regimen?

IV gamma globulin administration

The nurse is teaching a client newly diagnosed with a peanut allergy about how to manage the allergy. What information should be included in the teaching? Select all that apply. Identify ways to manage allergy while dining out. Food labels on baked items are the only labels that need to be read. Wear a medic alert bracelet. Carry EpiPen autoinjector at all times. List symptoms of peanut allergy.

Identify ways to manage allergy while dining out. Wear a medic alert bracelet. Carry EpiPen autoinjector at all times. List symptoms of peanut allergy.

What type of immunoglobulin does the nurse recognize that promotes the release of vasoactive chemicals such as histamine when a client is having an allergic reaction?

IgE

The nurse is teaching a group of health care workers about latex allergies. What reaction will the nurse teach the workers to be most concerned about with laryngeal edema?

IgE mediated hypersensitivity

A nurse notices a client lying on the floor at the bottom of the stairs. The client is alert and oriented and denies pain other than in the arm, which is swollen and appears deformed. After calling for help, what should the nurse do? Immobilize the client's arm. Place the client in a sitting position. Raise the client's arm above the heart. Help the client walk to the nearest nurses' station.

Immobilize the client's arm. Explanation: Signs of a fracture in an extremity include pain, deformity, swelling, discoloration, and loss of function. When a nurse suspects a fracture, the extremity should be immobilized before moving the body part. It isn't appropriate for the nurse to move the client into a sitting position without further assessment. The client shouldn't walk to the nurses' station; the client should wait for help to arrive.

A nurse in the emergency department is caring for a client who has a snakebite on her arm. Which of the following interventions should the nurse implement?

Immobilize the limb at the level of the heart. The emergency management of a client who has a snakebite focuses on limiting the spread of venom. any constrictive clothing or jewelry should be removed before the swelling worsens, and the affected limb should be immobilized at the level of the heart.

When writing a plan of care for a client with psoriasis, the nurse would know that an appropriate nursing diagnosis for this client would be what?

Impaired Skin Integrity Related to Scaly Lesions

An elderly client is diagnosed with a respiratory infection. While reviewing age-related changes in the immune system, what would the nurse identify as having contributed to this client's infection?

Impaired ciliary action from exposure to environmental toxins

A client presents in the emergency department with complaints of cough, headache, and generalized aches and pains. Upon assessment, the nurse documents a temperature of 102.5°F (39.2°C) and redness on the arms, legs, and upper chest. She also notes that the client takes eight different medications each day. What nursing diagnosis is the priority for this client?

Impaired tissue integrity

To reduce the incidence of complications in a client in traction, which intervention should be included in the care plan?

Increase fiber intake. Explanation Immobility increases the incidence of constipation. Increasing fiber intake will reduce GI complications. The weights in traction should never be removed. Inactivity results in fewer calories being burned. Increasing calories would be counterproductive. Reducing fluids will increase the likelihood of constipation.

A nurse understands the influence of hormones on bone maintenance. Therefore, the nurse knows that a patient on long-term cortisol may experience:

Increased bone resorption. Explanation: Increased levels of cortisol result in increased bone resorption and decreased bone formation. These patients are at increased risk for steroid-induced osteopenia and fractures.

Which assessment finding would cause the nurse to suspect compartment syndrome in the client following a bone biopsy?

Increased diameter of the calf Explanation: Increasing diameter of the calf can be indicative of bleeding into the muscle. The other findings are within normal limits.

Which nursing diagnosis takes highest priority for a client with a compound fracture?

Infection related to effects of trauma Explanation: A compound fracture involves an opening in the skin at the fracture site. Because the skin is the body's first line of defense against infection, any skin opening places the client at risk for infection. Imbalanced nutrition: Less than body requirements is rarely associated with fractures. Although Impaired physical mobility and Activity intolerance may be associated with any fracture, these nursing diagnoses don't take precedence because they aren't as life-threatening as infection.

The nurse is assessing a client with decreased dexterity of the hands related to rheumatoid arthritis. The nurse knows that which process causes joint deformities?

Inflammation Explanation: In clients with chronic inflammation, the immune response can deviate from normal. Instead of resolution of swelling and joint pain once the triggering event has subsided, pannus, or proliferation of newly formed synovial tissue infiltrated with inflammatory cells, formation occurs. Destruction of the joint's cartilage and erosion of bone soon follow. Remission is a period when the symptoms of the condition are reduced or absent. Exacerbation is a period when the symptoms occur or increase. Autoimmunity causes tissue destruction which leads to pain.

A female client is at risk for developing osteoporosis. Which action will reduce the client's risk?

Initiating weight-bearing exercise routines Explanation: Performing weight-bearing exercise increases bone health. A sedentary lifestyle increases the risk of developing osteoporosis. Estrogen is needed to promote calcium absorption. The recommended daily intake of calcium is 1,000 mg, not 300 mg.

A nurse is taking the health history of a newly admitted client and asks for a list of the client's current medications. Which medication classification would NOT place the client at risk for impaired immune function?

Inotropics

A 6-year-old client is diagnosed with a viral infection of the respiratory system. Which will most likely be trying to fight the antigen?

Interferons

The nurse is conducting a community education program on basal cell carcinoma (BCC). Which statement should the nurse make?

It begins as a small, waxy nodule with rolled translucent, pearly borders.

An older adult asks about a red papule that is on the right arm that loses color when pressure is applied. In which way will the nurse interpret this finding?

It is a cherry angioma that is a normal age-related skin alteration.

A client asks the nurse what psoriasis is. What is the best answer?

It is characterized by patches of redness covered with silvery scales.

A client has a serum study that is positive for the rheumatoid factor. What will the nurse tell the client about the significance of this test result?

It is suggestive of rheumatoid arthritis. Explanation: Rheumatoid factor is present in about 70% to 80% of patients with rheumatoid arthritis, but its presence alone is not diagnostic of rheumatoid arthritis, and its absence does not rule out the diagnosis. The antinuclear antibody (ANA) test is used to diagnose Sjögren's syndrome and systemic lupus erythematosus.

A client is being placed on a purine-restricted diet. What foods will the nurse include in the client's diet plan?

dairy products Explanation: Purines are chemical compounds found in high concentrations in certain foods, particularly shellfish, organ meats, and some alcoholic beverages (e.g., beer, distilled liquors). Purines are implicated in gout. Another reason that clients with gout are advised to avoid alcohol is that alcohol of any kind may increase the free fatty acid concentrations that can exacerbate gout.

Chronic illnesses may contribute to immune system impairment in various ways. Renal failure is associated with

deficiency in circulating lymphocytes.

A client with an allergic disorder calls the nurse and asks what treatment is available for allergic disorders. The nurse explains to the client that there is more than one treatment available. What treatments would the nurse tell the client about?

desensitization

During a routine checkup, a nurse assesses a client with acquired immunodeficiency syndrome (AIDS) for signs and symptoms of cancer. What is the most common AIDS-related cancer?

Kaposi's sarcoma

In a client infected with human immunodeficiency virus (HIV), CD4+ levels are measured to determine the:

extent of immune system damage.

A nurse planning care for a client who is to undergo a stem cell transplant. Which of the following actions should the nurse plan to take? Place the client in a negative airflow room Keep blood pressure equipment in the client's room Monitor the client's vital signs once every 8 hours Provide the client with 1,000 mL of water to drink ever 12 hours

Keep blood pressure equipment in the client's room The nurse should keep and use dedicated equipment, such as blood pressure monitor, stethoscope and thermometer in the client's room to prevent the spread of infection from client to client

A client undergoes open reduction with internal fixation to treat an intertrochanteric fracture of the right hip. The nurse should include which intervention in the postoperative care plan? Turning the client from side to side every 2 hours Keeping a pillow between the client's legs at all times Performing passive range-of-motion (ROM) exercises on the client's legs once each shift Maintaining the client in semi-Fowler's position

Keeping a pillow between the client's legs at all times Explanation: After open reduction with internal fixation, the client must keep the affected leg abducted at all times; placing a pillow between the legs reminds the client not to cross the legs and to keep the leg abducted. Passive or active ROM exercises shouldn't be performed on the affected leg during the postoperative period, because this could damage the operative site and cause hip dislocation. Most clients should be turned to the unaffected side, not from side to side. After open reduction with internal fixation, the client must avoid acute flexion of the affected hip to prevent possible hip dislocation; therefore, semi-Fowler's position should be avoided.

A nurse in a provider's office is assessing a client who has rheumatoid arthritis (RA). Which of the following findings is a late manifestation of this condition? Anorexia Knuckle deformity Low-grade fever Weight loss

Knuckle deformity Joint deformity is a late manifestation of RA

The nurse is performing an assessment on an older adult patient and observes the patient has an increased forward curvature of the thoracic spine. What does the nurse understand this common finding is known as?

Kyphosis Explanation: Common deformities of the spine include kyphosis, which is an increased forward curvature of the thoracic spine that causes a bowing or rounding of the back, leading to a hunchback or slouching posture. The second deformity of the spine is referred to as lordosis, or swayback, an exaggerated curvature of the lumbar spine. A third deformity is scoliosis, which is a lateral curving deviation of the spine (Fig. 40-4). Osteoporosis is abnormal excessive bone loss.

A client visits an orthopedic specialist because of pain beginning in the low back and radiating behind the right thigh and down below the right knee. The doctor suspects a diagnosis of sciatica. The nurse knows that the origin of the pain is between which intervertebral disks? L4, L5, and S1 C3, C4, and L1 L1, L2, and L4 L2, L3, and L5

L4, L5, and S1 Explanation: The lower lumbar disks, L4-L5 and L5-S1, are subject to the greatest mechanical stress and the greatest degenerative changes. Disk protrusion (herniated nucleus pulposus) or facet joint changes can cause pressure on nerve roots as they leave the spinal canal, which results in pain that radiates along the nerve.

A nurse is caring for a client who has a prescription for silver sulfadiazine cream to be applied to her burn wounds. The nurse should evaluate the client for which of the following laboratory findings?

Leukopenia Transient leukopenia is an adverse effect of silver sulfadiazine.

A protease inhibitor, an antiretroviral agent that can be taken without regard to meals is:

Lexiva

A patient has contact dermatitis on the hand, and the nurse observes an area that is thickened and rough between the thumb and forefinger. What does the nurse know that this is significant of related to repeated scratching and rubbing?

Lichenification

A community health nurse is providing teaching about malignant melanoma to a group of clients. The nurse should inform the group that which of the following traits places a client at risk for developing a malignant melanoma?

Light skin Light skin and less pigmentation place a client at risk for developing malignant melanoma.

A nurse is planning care for a client who has inmunosuppression following chemotherapy. Which of the following interventions should the nurse include in the plan of care? Insert an indwelling catheter to monitor sediment in the urine Take the client's temperature once per shift Provide the client with fresh fruit to avoid constipation Limit the number of health care workers entering the room

Limit the number of health care workers entering the room The nurse should limit the number of health care workers entering the client's room to prevent possible overexposure to microorganisms that can lead to an infection

Which option should the nurse encourage to replace fluid and electrolyte losses in a client with AIDS?

Liquids

A nurse is assessing a client who has a puncture wound on his foot. Which of the following findings is a manifestation of acute osteomyelitis?

Localized erythema Swelling and localized erythema are manifestations of acute osteomyelitis

The nurse is performing a physical assessment for a patient at the clinic and palpates enlarged inguinal lymph nodes on the left. What should the nurse document? (Select all that apply.) Location Size Consistency Reports of tenderness Temperature

Location Size Consistency Reports of tenderness

Assessment of a client reveals signs and symptoms of Paget's disease. Which of the following would be most likely?

Long bone bowing Explanation: Some clients with Paget's disease are asymptomatic with only some mild skeletal deformity. Other clients have marked skeletal deformities which may include enlargement of the skull, bowing of the long bones, and kyphosis.

The nurse teaches the client that reducing the viral load will have what effect?

Longer survival

Which of the following deformity causes a exaggerated curvature of the lumbar spine?

Lordosis Explanation: Lordosis is an exaggerated curvature of the lumbar spine. Scoliosis is a lateral curving deviation of the spine. Kyphosis is an increased forward curvature of the thoracic spine. Steppage gait is not a type of spinal deformity.

The client presents with an exaggeration of the lumbar spine curve. How does the nurse interpret this finding?

Lordosis Explanation: Lordosis is an exaggeration of the lumbar spine curve.

The nurse is employed at a long-term care facility caring for geriatric clients. Which assessment finding is characteristic of an age-related change?

Loss of height Explanation: A common age-related change is the loss of height due to the loss of bone mass and vertebral collapse. Cognitive decline is not an age-related change. Depression occurs in all age groups. Geriatric clients have a decrease in muscle mass.

Which of the following is a center for immune cell proliferation?

Lymph node

A client is diagnosed with human immunodeficiency virus (HIV). After recovering from the initial shock of the diagnosis, the client expresses a desire to learn as much as possible about HIV and acquired immunodeficiency syndrome (AIDS). When teaching the client about the immune system, the nurse states that humoral immunity is provided by which type of white blood cell?

Lymphocyte

The nursing students are learning about the immune system in their anatomy and physiology class. What would these students learn is a component of the immune system?

Lymphoid tissues

The nurse is aware that the phagocytic immune response, one of the body's responses to invasion, involves the ability of cells to ingest foreign particles. Which of the following engulfs and destroys invading agents?

Macrophages

A nurse is teaching a group of clients about osteoarthritis. Which of the following recommendations should the nurse include in the teaching? Use Echinacea to manage joint pain. Apply ice to the joint before exercising. Maintain a recommended body weight. Reduce the amount of purine in the diet.

Maintain a recommended body weight. Obesity is a risk factor for the development of osteoarthritis. Maintenance of an ideal weight is one way a client can prevent added wear and tear on joints and promote overall joint health.

A client with a fractured femur is in balanced suspension traction. The client needs to be repositioned toward the head of the bed. During repositioning, what should the nurse do? Maintain consistent traction tension while repositioning. Place slight additional tension on the traction cords. Release the weights and replace them immediately after positioning. Reposition the bed instead of repositioning the client.

Maintain consistent traction tension while repositioning. Explanation: Traction is used to reduce the fracture and must be maintained at all times, including during repositioning. It would be inappropriate to add tension or release the weights. Moving the bed instead of the client is not feasible.

A client with degenerative joint disease asks the nurse for suggestions to avoid unusual stress on the joints. Which suggestion would be most appropriate?

Maintain good posture. Explanation: The nurse needs to remind the client with degenerative joint disease to maintain good posture. While the client need not maintain complete bed rest, performing aerobic exercises is not advisable as it may place undue stress on the joint worsening the condition. Shifting weight from one foot to the other does not help avoid unusual stress on a joint.

After a traumatic back injury, a client requires skeletal traction. Which intervention takes priority?

Maintaining traction continuously to ensure its effectiveness Explanation: The nurse must maintain skeletal traction continuously to ensure its effectiveness. The nurse should assess skin for breakdown; however, maintaining skeletal traction takes priority. Traction weights must hang freely to be effective; they should never be supported. The nurse should increase, not restrict, the client's fluid and fiber intake (unless contraindicated by a concurrent illness) to prevent constipation associated with complete bed rest.

The nurse is preparing to perform a Wood's light examination. Which of the following would be most important for the nurse to do?

Make sure that the room is darkened.

A nurse is caring for a client who has a lesion on the back of his right hand. The client asks the nurse which type of skin cancer is the most serious. Which of the following responses by the nurse is appropriate?

Melanomas Melanomas are malignant neoplasms with atypical melanocytes in both the epidermis, the dermis, and sometimes the subcutaneous cells. It is the most lethal type of skin cancer, often causes metastases in the bone, liver, lungs, spleen, the CNS, and lymph nodes.

The nurse is discussing the new medication that a client will be taking for treatment of rheumatoid arthritis. Which disease-modifying antirheumatic drug (DMARD) will the nurse educate the client about?

Methotrexate Explanation: Methotrexate is a DMARD that reduces the amount of joint damage and slows the damage to other tissues as well. Celecoxib is a nonsteroidal anti-inflammatory drug (NSAID). Methylprednisolone is a steroid to reduce pain and inflammation and slow joint destruction. Mercaptopurine azathioprine is a cytotoxic drug.

A parent has brought a child to the clinic for a wellness check. While talking with the nurse, the parent asks the nurse to suggest a diet that will maximize the immune function of growing children. What dietary pattern should the nurse suggest?

Moderate diet that is balanced and varied

While examining a client's leg, a nurse notes an open ulceration with visible granulation tissue in the wound. Until a wound specialist can be contacted, which type of dressing should the nurse apply?

Moist sterile saline gauze

The nurse is working on an orthopedic floor caring for a client injured in a football game. The nurse is reviewing the client's chart noting that the client has previously had an injured tendon. The nurse anticipates an injury between the periosteum of the bone and which of the following?

Muscle Explanation: Tendons attach muscles to the periosteum of bone. Joints are a junction between two or more bones. Ligaments connect two freely movable bones. Cartilage is a dense connective tissue used to reduce friction between two structures.

A client scheduled to undergo an electromyography asks the nurse what this test will evaluate. What is the correct response from the nurse?

Muscle weakness Explanation: Electromyography tests the electric potential of the muscles and nerves leading to the muscles. It is done to evaluate muscle weakness or deterioration, pain, disability, and to differentiate muscle and nerve problems. A bone biopsy is done to identify bone composition. Bone densitometry is done to evaluate bone density. A bone scan would be appropriate to detect metastatic bone lesions.

A child is brought to the clinic with a rash and is subsequently diagnosed with measles. The parent reports also having had measles as a young child. What type of immunity to measles develops after the initial infection?

Naturally acquired active immunity

An adult client has had mumps when the client was a child. The client had a titer prior to entering nursing school and shows immunity. What type of immunity does this reflect?

Naturally acquired active immunity

The nurse is caring for a client with a furuncle. What advice should the nurse give a client with a furuncle to prevent the spread of the infection?

Never pick or squeeze a furuncle.

A nurse knows that a person with a 3-week-old femur fracture is at the stage where angiogenesis is occurring. What are the characteristics of this stage?

New capillaries producing a bridge between the fractured bones. Explanation: Angiogenesis and cartilage formation begin when fibroblasts from the periosteum produce a bridge between the fractured bones. This is known as a callus.

The nurse is caring for a client with ankylosing spondylitis (AS). Which medication will the nurse expect to be prescribed for this client?

Nonsteroidal anti-inflammatory drugs (NSAIDs) NSAIDS are the first-line therapy for treating all spondyloarthropathies. Antibiotics and anticoagulants are not used to treat AS. Corticosteroid injections may be used for periodic flares; however, oral and long-term use of steroids is not recommended.

A nurse is admitting a client to the unit who presented with a lower extremity fracture. What signs and symptoms would suggest to the nurse that the client may have a peroneal nerve injury? Numbness and burning of the foot Visible cyanosis in the toes Pallor to the dorsal surface of the foot Inadequate capillary refill to the toes

Numbness and burning of the foot Explanation: Peroneal nerve injury may result in numbness, tingling, and burning in the feet. Cyanosis, pallor, and decreased capillary refill are signs of inadequate circulation

A female client comes to the clinic and tells the nurse, "I think I have another vaginal infection and I also have some wart-like lesions on my vagina. This is happening quite often." Which nursing action is the priority for this client?

Offer information on human immunodeficiency virus (HIV) testing.

A client has had a "stuffy nose" and obtained an oxymetazoline nasal spray. What education should the nurse provide to the client in order to prevent "rebound congestion"?

Only use the nasal spray for 3 to 4 days once every 12 hours.

A client with carpal tunnel syndrome has had limited improvement with the use of a wrist splint. The nurse knows that which procedure will show the greatest improvement in treatment for this client? Laser therapy Open nerve release Ultrasound therapy Injection of lidocaine

Open nerve release Explanation: Evidence-based treatment of acute carpal tunnel syndrome includes the application of splints to prevent hyperextension and prolonged flexion of the wrist. Should this treatment fail, open nerve release is a common surgical management option. A variety of treatments may be tried by the client, however, they may fail to improve the condition. These treatments include laser therapy, ultrasound therapy, and the injection of substances such as lidocaine. Though these can be used, surgery to release nerves is the best option.

An instructor is describing the process of bone development. Which of the following would the instructor describe as being responsible for the process of ossification?

Osteoblasts Explanation: Osteoblasts secrete bone matrix (mostly collagen), in which inorganic minerals, such as calcium salts, are deposited. This process of ossification and calcification transforms the blast cells into mature bone cells, called osteocytes, which are involved in maintaining bone tissue. Cortical bone is dense hard bone found in the long shafts; cancellous bone is spongy bone found in the irregular rounded edges of bone.

A client is informed of having a benign bone tumor but that this type of tumor that may become malignant. The nurse knows that this is characteristic of which type of tumor? Osteoid osteoma Enchondroma Osteoclastoma Osteochondroma

Osteoclastoma Explanation: An osteoclastoma is a giant cell tumor that may invade local tissue; usually soft and hemorrhagic and may become malignant. An osteochondroma occurs as a large projection of bone at the ends of long bones, developing during growth periods and then becoming static bone mass. An enchondroma is a hyaline cartilage tumor that develops in the hand, ribs, femur, tibia, humerus, or pelvis. An osteoid osteoma is a painful tumor surrounded by reactive bone tissue.

A group of students are reviewing information about bones in preparation for a quiz. Which of the following indicates that the students have understood the material?

Osteoclasts are involved in the destruction and remodeling of bone. Explanation: Osteoclasts are the cells involved in the destruction, resorption, and remodeling of bone. Red bone marrow is responsible for manufacturing red blood cells. Long bones contain yellow bone marrow; the sternum, ileum, vertebrae, and ribs contain red bone marrow. Osteoblasts are transformed into osteocytes, mature bone cells.

Which of the following is the most common and most fatal primary malignant bone tumor? Rhabdomyoma Osteogenic sarcoma (osteosarcoma) Enchondroma Osteochondroma

Osteogenic sarcoma (osteosarcoma) Explanation: Osteogenic sarcoma (osteosarcoma) is the most common and most often fatal primary malignant bone tumor. Benign primary neoplasms of the musculoskeletal system include osteochondroma, enchondroma, and rhabdomyoma.

A client presents to the emergency department with fever, chills, restlessness, and limited movement of a fractured jaw. What complication should the nurse interpret as the findings? Fat embolism Osteomyelitis Compartment syndrome Avascular necrosis

Osteomyelitis Explanation: Clinical manifestations of osteomyelitis include signs and symptoms of sepsis and localized infection. A client with avascular necrosis does not have fever and chills. Clients with fat emboli will have a rash and breathing complications. A client with compartment syndrome will have numbness, not a fever.

A nurse is collecting a health history from a client. Which of the following findings is the highest risk factor for the client developing skin cancer?

Overexposure to sun light The nurse should apply the safety and risk reduction priority-setting framework when caring for this client. This framework assigns priority to the factor or situation posing the greatest safety risk to the client. When there are several risks to client safety, the one posing the greatest threat is the highest priority. The nurse should use Maslow's hierarchy of needs, the ABC priority setting framework, or nursing knowledge to identify which risk poses the greatest threat to the client. Therefore, the nurse should identify the client's overexposure to sun as being the greatest risk factor for developing skin cancer.

Which is a hallmark sign of compartment syndrome?

Pain Explanation: A hallmark sign of compartment syndrome is pain that occurs or intensifies with passive range of motion

A client is having patch testing performed to identify allergies to substances. Which finding most concern the nurse? Select all that apply. Pain Itching Blisters Papules Redness

Pain Blisters

Which of the following is a strong positive reaction to patch testing? Select all that apply. Itching Pain Ulceration Redness Blisters

Pain Ulceration Blisters

A nurse is caring for a client who is receiving chemotherapy to treat cancer. Which of the following adverse effects should the nurse anticipate from the chemotherapy?

Pancytopenia Pancytopenia, a deficiency of WBCs, RBCs, and platelet counts, is an expected adverse effect of chemotherapy.

A nurse in a provider's office is assessing a client's skin lesions. The nurse notes that the lesions are 0.5 cm (0.20 in) in size, elevated, and solid, with very distinct borders. The nurse should document the findings as which of the following skin lesions?

Papules A papule is a small, solid, elevated lesion with distinct borders. It is usually smaller than 10 mm in diameter. Papules are common lesions of wart and elevated moles.

Which of the following are the major hormonal regulators of calcium homeostasis? Select all that apply. Parathormone Calcitonin Thyroid Cortisol Growth hormone

Parathormone Calcitonin

The nurse observes a client with a shuffling gait. What disease is commonly associated with a shuffling gait?

Parkinson's disease Explanation: Client with Parkinson's disease may have a shuffling gait. Clients with a lower motor disease will have steppage gait. Clients with scoliosis may have a limp. Clients with Paget's disease may have bone fractures.

A laboring mother asks the nurse if the baby will have immunity to some illnesses when born. What type of immunity does the nurse understand that the newborn will have?

Passive immunity transferred by the mother

A nurse knows that more than 50% of clients with CVID develop the following disorder.

Pernicious anemia

The client presents with nausea and vomiting, sluggish bowel sounds, and abdominal distention. How does the nurse interpret these findings? Disuse syndrome Impaired physical mobility Psychological cast syndrome Physiologic cast syndrome

Physiologic cast syndrome Explanation: Physiological cast syndrome is characterized by impaired gastrointestinal function, such as nausea and vomiting, sluggish bowel sounds, and abdominal distention

A nurse is caring for a client who is postoperative following a total hip arthroplasty. The nurse assists the client into a supine position. Which of the following actions is appropriate to prevent dislocation of the hip? Place a wedge pillow between the legs. Elevate the head of the bed to a Fowler's position. Position the legs in alignment with the spine. Place a footboard on the bed.

Place a wedge pillow between the legs. The nurse places a wedge pillow or other abduction device between the legs to prevent adduction which can lead to possible dislocation.

The nurse completes a history and physical assessment on a client with acquired immune deficiency syndrome (AIDS) who was admitted to the hospital with respiratory complications. The nurse knows to assess for what common infection (80% occurrence) in persons with AIDS?

Pneumocystis pneumonia

A client is diagnosed with severe combined immunodeficiency (SCID). What would the nurse expect to integrate into the client's plan of care?

Preparation for bone marrow transplantation

A nurse is planning care for a client who is postoperative following a total hip arthroplasty. Which of the following interventions should the nurse include in the plan of care? Instruct the client to avoid movement of the affected leg Prevent hip flexion of the affected extremity Position the lower extremities so that they are touching Ensure that the client's heels are touching the bed

Prevent hip flexion of the affected extremity The nurse should implement measures to prevent hip flexion of the affected extremity beyond 90 degrees due to the risk of dislocation. Raised toilet seats and reclining chairs help prevent hyper-flexion.

A patient had unprotected sex with an HIV-infected person and arrives in the clinic requesting HIV testing. Results determine a negative HIV antibody test and an increased viral load. What stage does the nurse determine the patient is in?

Primary infection

The nurse is caring for a client with an autoimmune disease. What is a characteristic of autoimmune disorders?

Progressive tissue damage without any verifiable etiology

During which stage of the immune response does the circulating lymphocytes containing the antigenic message return to the nearest lymph node?

Proliferation

The nurse is caring for a client who underwent a total hip replacement yesterday. What should the nurse do to prevent dislocation of the new prosthesis? Have the client reposition himself independently. Protect the affected leg from internal rotation. Keep the affected leg in a position of adduction. Keep the hip flexed by placing pillows under the client's knee.

Protect the affected leg from internal rotation. Explanation: Abduction of the hip helps to prevent dislocation of a new hip joint. Rotation and adduction should be avoided. While the hip may be flexed slightly, it shouldn't exceed 90 degrees and maintenance of flexion isn't necessary. The client may not be capable of safe independent repositioning at this early stage of recovery.

A nurse is caring for a client who has had an allogeneic hematopoietic stem-cell transplant. Which of the following infection-control precautions should the nurse use while caring for this client? Airborne Protective Contact Droplet

Protective Clients whose immune system is compromised, such as from chemotherapy, AIDS, or after a stem-cell transplant, require a protective environment. ———————————————————————————— Clients who have varicella and other infections such as rubeola and tuberculosis require airbone precautions. Clients who have infections such as herpes simplex, respiratory syncyial virus, and methicillin-resistant Staphylococcus aureus require contact precautions. Clients who have streptococcal pharyngitis and other infections such as rubella and diphtheria require droplet precautions

A client with a traumatic amputation of the right lower leg is refusing to look at the leg. Which action by the nurse is most appropriate? Request a referral to occupational therapy. Provide wound care without discussing the amputation. Provide feedback on the client's strengths and available resources. Encourage the client to perform range-of-motion (ROM) exercises to the right leg.

Provide feedback on the client's strengths and available resources. Explanation: The nurse should encourage the client to look at, and assist with, care of the residual limb. Providing feedback on the client's strengths and resources may allow the client to start to adapt to the body image and lifestyle change. The nurse should also allow time for the client to discuss their feelings related to the amputation. Requesting a referral to occupational therapy and encouraging the client to perform ROM exercises are appropriate but do not address the emotional aspect of losing an extremity.

A nurse is caring for a client who ha pruritus following treatment for scabies. Which of the following actions should the nurse take? Apply additional scabicide to the affected area Assist the client to take a hot shower Provide mittens for the client to wear at night Encourage the client to gently rub the affected area

Provide mittens for the client to wear at night Intense itching is a manifestation of scabies that is often reported by clients as unbearable at night. For this reason, the nurse should provide mittens for the client to wear at night to protect the integrity of the skin

A client is undergoing photochemotherapy involving a combination of a photosensitizing chemical and ultraviolet light. What health problem does this client most likely have?

Psoriasis

A patient is diagnosed with psoriasis after developing scales on the scalp, elbows, and behind the knees. The patient asks the nurse where this was "caught." What is the best response by the nurse?

Psoriasis is an inflammatory dermatosis that results from an overproduction of keratin.

The nurse is caring for a patient who had a total hip replacement. What lethal postoperative complication should the nurse closely monitor for? Hypovolemia Urinary tract infection Atelectasis Pulmonary embolism

Pulmonary embolism Explanation: Patients having orthopedic surgery are particularly at risk for venous thromboembolism, including deep vein thrombosis and pulmonary embolism.

Primary gout is often caused by an inherited disorder in which type of metabolism?

Purine Explanation: Primary gout is often caused by an inherited disorder in purine metabolism. Primary gout is not a disorder of altered carbohydrate, fat, or glucose metabolism.

A client arrives at the orthopedic clinic and reports suspecting a stress fracture of the right foot. The physician orders an x-ray with negative results. What does the nurse understand that these negative results can mean?

Purine-rich foods Clients with gout should be advised to have adequate protein with the limitation of purine-rich foods to avoid contributing to the underlying problem. The diet should also be relatively high in carbohydrates and low in fats because carbohydrates increase urate excretion and fats retard it. A high fluid intake is recommended because it helps increase the excretion of uric acid.

The nurse is assessing the periwound skin of an African American client for inflammation. The nurse determines that inflammation is present when which characteristic is noted?

Purple-gray cast

When assessing a patient with risk factors related to human immunodeficiency virus (HIV), what does the nurse know can be the first manifestation of the disease?

Purplish cutaneous lesions

A patient's skin is examined and the nurse notes the presence of herpes simplex/zoster skin lesions. The nurse describes the lesions as:

Pus-filled vesicles; circumscribed and elevated masses >0.5 cm.

The nurse is educating the patient with low back pain about the proper way to lift objects. What muscle should the nurse encourage the patient to maximize?

Quadriceps Explanation: The nurse instructs the patient in the safe and correct way to lift objects using the strong quadriceps muscles of the thighs, with minimal use of weak back muscles (Fig. 42-3).

The community health nurse is assessing the risk factors for osteoporosis in a female client at a health fair. For each assessment finding, click to specify if the finding is a risk factor for osteoporosis or is not a risk factor for osteoporosis. alcohol intake of 3 drinks/week takes fluticasone inhaler for asthma large frame Nonsmoker postmenopausal status Asian heritage walks 2 miles, 3 days/week 66 years of age

RF NRF Asian heritage large frame 66 years of age nonsmoker Postmenopausal status alcohol intake of 3 drinks/week takes fluticasone inhaler walks 2 miles, 3 days/week for asthma

Which term refers to a disease of a nerve root? Contracture Radiculopathy Involucrum Sequestrum

Radiculopathy Explanation: When the client reports radiating pain down the leg, the client is describing radiculopathy. Involucrum refers to new bone growth around the sequestrum. Sequestrum refers to dead bone in an abscess cavity. Contracture refers to abnormal shortening of muscle or fibrosis of joint structures.

A nurse is planning care for a client who has cancer and has developed thrombocytopenia following chemotherapy. Which of the following precautions should the nurse offer to minimize the adverse effect of thrombocytopenia?

Remind the client to use an electric razor Thrombocytopenia is a decrease in the client's blood platelet count, which places the client at an increased risk of bleeding due to the blood's inability to clot. Therefore, the nurse should institute bleeding precautions, which includes the use of an electric razor

A client with Crohn's disease, an autoimmune disorder, informs the nurse about not having any symptoms of the disease in 8 months. What does the nurse understand this asymptomatic period is referred to

Remission

A client with a history of anaphylactic reactions to insect stings has just been stung by a wasp. Place the steps in the correct order that the client will follow for self-administration of an EpiPen. Use all options.

Remove the gray safety-release cap Inject the black tip into the outer thigh Massage the injection area Call the emergency medical response number (911)

A client has just undergone arthrography. What is the most important instruction for the nurse to include in the teaching plan?

Report joint crackling or clicking noises occurring after the second day. Explanation: After undergoing arthrography, the client must be informed that he or she may hear crackling or clicking noises in the joints for up to 2 days, but if noises occur beyond this time, they should be reported. These noises may indicate the presence of a complication, and therefore should not be ignored or treated by the client. Massage is not indicated. The client need not be asked to avoid sunlight or dairy products.

A nurse is caring for a client who has contact dermatitis of the neck and upper chest. Which of the following is an expected finding?

Report of exposure to a skin irritant The most common cause of contact dermatitis is exposure to a topical irritant therefore identifying this irritant is a component of treatment

The nurse administers an injection to a client with AIDS. When finished, the nurse attempts to recap the needle and sustains a needlestick to the finger. What is the priority action by the nurse?

Report the incident to a supervisor

A client taking abacavir has developed fever and rash. What is the priority nursing action?

Report to the health care provider.

Which adverse effect(s) should the nurse closely monitor in a client who has secondary immunodeficiencies due to immunosuppressive therapy?

Respiratory or urinary system infections

A client with HIV will be started on a medication regimen of three medications. What class of drugs will the nurse instruct the client about?

Reverse transcriptase inhibitors

The nurse has been assigned to care for a client with a wound. Based on the principles of wound care, the nurse should proceed in the following sequential order:

Review clinical indications, followed by select materials, followed by ____, followed by reassess dressing materials, and then by ____

A client is brought to the emergency department by a softball team member who states the client and another player ran into each other, and the client is having severe pain in the right shoulder. What symptoms of a fractured clavicle does the nurse recognize? Right shoulder slopes downward and droops inward. Right shoulder is elevated above the left. Client complains of pain in the unaffected shoulder. Client complains of tingling and numbness in the right shoulder.

Right shoulder slopes downward and droops inward. The client with a fractured clavicle has restricted motion, and the affected shoulder appears to slope downward and droop inward. The client will have pain, not typically tingling and numbness in the right shoulder. Pain is not felt in the unaffected shoulder.

A nurse is providing teaching to a client who has asthma and a new prescription for inhaled beclomethasone. Which of the following instructions should the nurse provide? Check the pulse after medication administration Take the medication with meals Rinse the mouth after administration Limit caffeine intake

Rinse the mouth after administration. Use of glucocorticoids by metered dose inhaler can allow a fungal overgrowth in the mouth. Rinsing the mouth after administration can lessen the likelihood of this complication

Which connective tissue disorder is characterized by insoluble collagen being formed and accumulating excessively in the tissues?

Scleroderma Explanation: Scleroderma occurs initially in the skin but also occurs in blood vessels, major organs, and body systems, potentially resulting in death. Rheumatoid arthritis results from an autoimmune response in the synovial tissue, with damage taking place in body joints. SLE is an immunoregulatory disturbance that results in increased autoantibody production. In polymyalgia rheumatic, immunoglobulin is deposited in the walls of inflamed temporal arteries.

With repeated reactions of contact dermatitis, which of the following can occur?

Secondary bacterial infection

A nurse in a provider's office is collecting a health history from a client who is at risk for primary osteoporosis. Which of the following findings is a risk factor for the development of osteoporosis?

Sedentary lifestyle A sedentary lifestyle places the client at risk for osteoporosis. Regular, weight-bearing exercises help to build bone tissue

A patient with a history of allergies comes to the clinic for an evaluation. The following laboratory test findings are recorded in a patient's medical record: Total serum IgE levels: 2.8 mg/mL White blood cell count: 5,100/cu mm Eosinophil count: 4% Erythrocyte sedimentation rate: 20 mm/h The nurse identifies which result as suggesting an allergic reaction?

Serum IgE level

A nurse is reviewing laboratory values for a client who has systemic lupus erythematosus (SLE). Which of the following values should give the nurse the best indication of the client's renal function? Serum creatinine Blood urea nitrogen (BUN) Serum sodium Urine-specific gravity

Serum creatinine A renal function disorder reduces the excretion of creatinine, resulting in increased levels of blood creatinine. Creatinine is a specific and sensitive indicator of renal function

A client asks a nurse, "What can I use to decrease my risk of exposure to HIV?" What will the nurse include as effective in reducing the risk of HIV exposure? Select all that apply. Lambskin condoms Sexual abstinence Latex male condoms Polyurethane female condoms Dental dams

Sexual abstinence Latex male condoms Polyurethane female condoms Dental dams

The school nurse is instructing a parent in the care and elimination of lice from their child's hair. The parent brings all of the products for care in a bag. Which contents are not appropriate for use?

Shampoo and conditioner

A nurse is providing teaching to a client who is wheelchair-bound and his caregiver about ways to reduce the risk of pressure ulcer formation. Which of the following instructions should the nurse include?

Shift your weight in the wheelchair every 15 minutes. This response addresses the safety issue of pressure ulcer risk. Pressure ulcers are most likely to develop if the client does not shift position frequently to relieve pressure.

A nurse is reviewing the dietary history of a client who has experienced anaphylaxis. What would the nurse identify as a common cause of anaphylaxis? Select all that apply. Chicken Shrimp Milk Beef Eggs

Shrimp Milk Eggs

A client with rheumatoid arthritis has infiltration of the lacrimal and salivary glands with lymphocytes as a result of the disease. What does the nurse understand that this clinical manifestation is?

Sicca syndrome Explanation: Sicca syndrome is a condition of dry eyes and dry mouth that can result from infiltration of the lacrimal and salivary glands with lymphocytes. Episcleritis is an inflammatory condition of the connective tissue between the sclera and conjunctiva. Glaucoma results from increased intraocular pressure, and cataracts are a clouding of the lens in the eye.

An elderly client's hip joint is immobilized prior to surgery to correct a femoral head fracture. What is the nurse's priority assessment? Signs of neurovascular compromise The presence of leg shortening The client's complaints of pain The presence of internal or external rotation

Signs of neurovascular compromise Explanation: Because impaired circulation can cause permanent damage, neurovascular assessment of the affected leg is always a priority assessment. Leg shortening and internal or external rotation are common findings with a fractured hip. Pain, especially on movement, is also common after a hip fracture.

A nurse in the outpatient clinic is assessing a client who has psoriasis. The nurse should expect which of the following findings? Unilateral lesions Serous drainage Intense pain Silvery, white scales

Silvery, white scales The characteristic lesions of psoriasis are thick, erythematous plaques covered by silvery scales

Biopsies are performed on which of the following? Select all that apply. Skin nodules Blisters Ulcers Plaques Keloids

Skin nodules Blisters Ulcers Plaques

A group of students are studying for an examination on joints. The students demonstrate understanding of the material when they identify which of the following as an example of a synarthrodial joint?

Skull at the temporal and occipital bones Explanation: A synarthrodial joint is immovable and can be found at the suture line of the skull between the temporal and occipital bones. Amphiarthrodial joints are slightly moveable and are found between the vertebrae. The finger and hip joints are examples of diarthrodial joints that are freely moveable.

A nurse in a clinic is assessing a client. Which of the following findings should the nurse identify as a risk factor for osteoporosis?

Smokes 1 pack of cigarettes per day The nurse should identify active or passive smoking as a risk factor for osteoporosis

A client with systemic lupus erythematosus (SLE) is prescribed hydroxychloroquine. Which teaching will the nurse include for this client?

Smoking cessation An antimalarial medication, hydroxychloroquine, is effective for managing cutaneous, musculoskeletal, and mild systemic features of SLE. However, smoking inhibits the effectiveness of hydroxychloroquine. Because of this, teaching on smoking cessation would be a priority. Teaching about exercise would not be a priority because of the medication. A high-protein diet is not indicated as treatment for SLE. Vitamin D supplements would be applicable if the client is taking corticosteroids.

Which is a strategy for lowering risk for osteoporosis?

Smoking cessation Explanation: Risk-lowering strategies include increased dietary calcium and vitamin D intake, smoking cessation, alcohol and caffeine consumption in moderation, and outdoor activity. Individual risk factors include low initial bone mass and increased age. A lifestyle risk factor is a diet low in calcium and vitamin D.

A nurse is evaluating the laboratory values of a client who is in the resuscitation of phase following a major burn. Which of the following laboratory findings should the nurse expect?

Sodium 132 mEq/L This laboratory finding is below the expected reference range. The nurse should anticipate a low sodium level because sodium is trapped in interstitial space. The normal sodium level is 135-145 mEq/L

The nurse assesses a client who is diagnosed with human immunodeficiency virus (HIV) for adverse reactions associated with the prescribed medication, abacavir. The nurse provides emergency intervention when the client exhibits the following symptoms:

Sore throat Cough Dyspnea

The clinic nurse is caring for a client with an injured body part that does not require rigid immobilization. What method of immobilization would the nurse expect the health care provider to use on a short-term basis? Cast Skin traction Brace Splint

Splint Explanation: A splint immobilizes and supports an injured body part in a functional position and is used when the condition does not require rigid immobilization, causes a large degree of swelling, or requires special skin treatment. Casts and traction provide rigid immobilization. A brace provides support, controls movement, and prevents additional injury for more long-term use.

Which device is designed specifically to support and immobilize a body part in a desired position? Splint Traction Sling Brace

Splint Explanation: A splint may be applied to a fractured extremity initially until swelling subsides. Splints are designed to provide stability for fractures that are unstable and to immobilize and support the body part in a functional position. A brace is an externally applied device to support a body part, control movement, and prevent injury; braces are used to enhance movement while preventing injury. A sling is a bandage used to support an arm temporarily while the client ambulates; it is not designed to immobilize the body part. Traction is the use of a pulling force on a body part and thus it is not designed to immobilize; the goal of traction is to achieve or maintain alignment, decrease muscle spasms and pain, or correct or prevent deformities.

The nurse is assessing the client for scoliosis. What will the nurse have the client do to perform the assessment?

Stand behind the client and ask the client to bend forward at the waist. Explanation: Scoliosis is characterized by a lateral curvature of the spine. The nurse stands behind the client and asks the client to bend forward at the waist for the nurse to examine the spine curvature. The nurse cannot see the spine by standing beside the client or in front of the client. The spinal curve cannot be seen by watching the client walk.

The nurse is completing a health history with a client diagnosed with systemic lupus erythematosus (SLE). Which information will the nurse identify as environmental triggers for the condition? Select all that apply. Vegetarian diet Stress Sunlight Recent surgery Cigarette smoking

Stress Sunlight Recent surgery Cigarette smoking

A nurse is teaching a community group about healthy lifestyles. A participant asks about how to maintain a healthy immune system. The nurse informs the group that which factor will positively affect the immune system?

Strong family and community connections

A nurse practitioner assesses a patient's movement in his left hand after a cast is removed. The nurse asks the patient to turn his wrist so the palm of his hand is facing up. This movement is known as:

Supination. Explanation: Refer to Figure 40-3 in the text for an illustration of body movements produced by muscle contraction.

A client with chronic osteomyelitis has undergone 6 weeks of antibiotic therapy. The wound appearance has not improved. What action would the nurse anticipate to promote healing? Wound irrigation Wound packing Vitamin supplements Surgical debridement

Surgical debridement Explanation: In chronic osteomyelitis, surgical debridement is used when the wound fails to respond to antibiotic therapy. Wound packing, vitamin supplements, and wound irrigation are not the standard of care when treating chronic osteomyelitis.

A nurse is planning care for a client who has been admitted for treatment of a malignant melanoma of the upper leg without metastasis. The nurse should plan to prepare the client for which of the following procedures?

Surgical excision The therapeutic approach to malignant melanoma depends on the level of invasion and the depth of the lesion. Surgical excision is the treatment of choice for small, superficial lesions. Deeper lesions require wide local excision, followed by skin grafting.

Morton neuroma is exhibited by which clinical manifestation? Longitudinal arch of the foot is diminished Inflammation of the foot-supporting fascia Swelling of the third (lateral) branch of the median plantar nerve High arm and a fixed equinus deformity

Swelling of the third (lateral) branch of the median plantar nerve Explanation: Morton neuroma is swelling of the third branch of the median plantar nerve. Pes cavus refers to a foot with an abnormally high arch and a fixed equinus deformity of the forefoot. Flatfoot is a common disorder in which the longitudinal arch of the foot is diminished. Plantar fasciitis is an inflammation of the foot-supporting fascia.

Skull sutures are an example of which type of joint?

Synarthrosis Explanation: Skull sutures are considered synarthrosis joints and are immovable. Amphiarthrosis joints allow limited movement, such as a vertebral joint. Diarthrosis joints are freely movable joints such as the hip and shoulder. Aponeuroses are broad, flat sheets of connective tissue.

T-cell and B-cell lymphocytes are the primary participants in the immune response. What do they do?

T-cell and B-cell lymphocytes distinguish harmful substances and ignore those natural and unique to a person.

The anatomy and physiology instructor is explaining a cell-mediated response to the pre-nursing students. What actions would the instructor explain occur in a cell-mediated response?

T-cell lymphocytes survey proteins in the body and attack the invading antigens.

A nurse is performing discharge teaching for an elderly client with osteoporosis. Which instruction about a calcium supplement should the nurse include? Take weekly on the same day and at the same time. Remain in an upright position 30 minutes after taking the supplement. Take the supplement on an empty stomach with a full glass of water. Take the supplement with meals or with orange juice.

Take the supplement with meals or with orange juice. Explanation: Calcium supplements should be taken with meals or with a beverage high in vitamin C for increased absorption. Calcium supplements are taken daily, not weekly. There are no special instructions about staying upright when taking calcium supplements.

A client taking fosamprenavir reports "getting fat." What is the nurse's best action?

Teach the client about medication side effects.

A client's left leg is in skeletal traction with a Thomas leg splint and Pearson attachment. Which intervention should the nurse include in this client's care plan? Assess the client's level of consciousness. Apply the traction straps snugly. Teach the client how to prevent problems caused by immobility. Remove the traction at least every 8 hours.

Teach the client how to prevent problems caused by immobility. Explanation: By teaching the client about prevention measures, the nurse can help prevent problems caused by immobility, such as hypostatic pneumonia, muscle contracture, and atrophy. The nurse applies traction straps for skin traction — not skeletal traction. For a client in skeletal traction, the nurse should assess the affected limb, rather than assess the level of consciousness. Removing skeletal traction is the physician's responsibility — not the nurse's.

The nurse working in the orthopedic surgeon's office is asked to schedule a shoulder arthrography. The nurse determines that the surgeon suspects which finding?

Tear in the joint capsule Explanation: Arthrography is useful in identifying acute or chronic tears of the joint capsule or supporting ligaments of the knee, shoulder, ankle, hip, or waist. X-rays are used to diagnose bone fractures. Bone densitometry is used to estimate bone mineral density. An electromyogram (EMG) provides information about the electrical potential of the muscles and nerves leading to them.

A nurse is assessing a client who sustained superficial partial-thickness and deep partial-thickness burns 72 hours ago. Which of the following findings should the nurse report to the provider?

Temperature of 39.1° C (102.4° F) An elevated temperature is an indication of infection and the nurse should report this finding to the provider. Sepsis is a critical finding following a major burn injury. Initially, burn wounds are relatively pathogen-free. On approximately the third day following the injury, early colonization of the wound surface by gram-negative organisms changes to predominantly gram-positive opportunistic organisms.

A parent brings a young child to the clinic for an evaluation of an infection. The parent states, "my child has been taking antibiotics now for more than 2 months and still doesn't seem any better." During the history and physical examination, what would alert the nurse to suspect a primary immunodeficiency?

Ten ear infections in the past year

The nurse is gathering data from laboratory studies for a client who has HIV. The client's CD4+ cell count is 200/mm³, and the client has been diagnosed with pneumocystis pneumonia. What does this indicate to the nurse?

The client has converted from HIV infection to AIDS.

A client is diagnosed with a first-degree strain of the left ankle related to running 5 miles daily. How would the nurse differentiate the first-degree strain from other strains and sprains? The client is unable to bear weight on the left ankle and has a large ecchymotic area. The client complains of pain when the joint is moved and has mild edema. The client has some edema of the left ankle with muscle spasms but is able to walk without assistive devices. The client has ecchymosis, edema, and has no function of the left foot and ankle.

The client has some edema of the left ankle with muscle spasms but is able to walk without assistive devices. Explanation: A first-degree strain involves mild stretching of the muscle or tendon, causing some edema and muscle spasm, but no real loss of function. The second-degree strain is partial tearing of muscle or tendon, leading to inability to bear weight and causing edema, muscle tenderness, muscle spasm, and ecchymosis. The third-degree tear is severe muscle and/or tendon tearing, causing severe pain, muscle spasm ecchymosis, edema, and loss of function. A first-degree sprain involves stretching of the ligament fibers characterized by mild edema, tenderness, and pain if the joint is moved.

A nurse is collecting a health history from a client. Which of the following findings is the highest risk factor for the client developing bladder cancer?

The client uses tobacco The nurse should apply the safety and risk reduction priority-setting framework when caring for this client. This framework assigns priority to the factor or situation posing the greatest safety risk to the client. When there are several risks to client safety, the one posing the greatest threat is the highest priority. The nurse should use Maslow's hierarchy of needs, the ABC priority setting framework, or nursing knowledge to identify which risk poses the greatest threat to the client. Therefore, the nurse should identify the client's tobacco use as being the greatest risk factor for developing bladder cancer.

Which client(s) is most likely to have compartment syndrome after sustaining a fracture? Select all that apply. The client with a plaster cast applied immediately after injury The client with elevated pressure within the muscles The client with hemorrhage in the site of injury The client using ice to control pain in the extremity The client who sustained a clavicle fracture

The client with a plaster cast applied immediately after injury The client with elevated pressure within the muscles The client with hemorrhage in the site of injury Compartment syndrome occurs in cases of fracture when the normal pressure of a compartment is altered by the force of the injury itself, by development of edema, or by hemorrhaging at the site of the injury, which increases the contents of the compartment, or from outside pressure caused by constriction from a dressing or cast. A client with elevated muscle pressure is at risk for compartment syndrome. The application of a plaster cast immediately after the injury places the client at risk for compartment syndrome because the cast will not allow for edema and therefore will compress the tissue. Clavicle fractures are not a risk factor for compartment syndrome because of the location of the fracture. Ice will assist in decreasing edema and may help prevent compartment syndrome.

A client has been diagnosed with AIDS and tuberculosis (TB). A nursing student asks the nurse why the client's skin test for TB is negative if the client's physician has diagnosed TB. The nurse's correct reply is which of the following?

The client's immune system cannot mount a response to the skin test.

A client has begun to suffer from rheumatoid arthritis and is being assessed for disorders of the immune system. The client works as an aide at a facility that cares for children infected with AIDS. What is the most important factor related to the client's assessment?

The client's use of other drugs

Which is not a guideline for avoiding hip dislocation after replacement surgery. Never cross the legs when seated. The hip may be flexed to put on clothing such as pants, stockings, socks, or shoes. Keep the knees apart at all times. Put a pillow between the legs when sleeping.

The hip may be flexed to put on clothing such as pants, stockings, socks, or shoes. Explanation: Guidelines for avoiding hip dislocation after replacement surgery specify that the hip should not be flexed to put on clothing such as pants, stockings, socks, or shoes. Clients should keep the knees apart at all times, put a pillow between the legs when sleeping, and never cross the legs when seated.

A nurse is caring for a client with multiple sclerosis. Client education about the disease process includes which explanation about the cause of the disorder?

The immune system recognizes one's own tissues as "foreign."

A nurse is reviewing the causes of genetic diseases with parents of an infant born with severe combined immunodeficiency disease (SCID). Which would be inaccurate information pertaining to SCID?

The inheritance of SCID can be autosomal dominant.

The nurse is administering a skin test for detection of exposure to tuberculosis. How would the nurse determine if the client was exposed to tuberculosis?

The injection area swells if the client has developed antibodies against the antigen.

The nurse is conducting a community education program on allergies and anaphylactic reactions. The nurse determines that the participants understand the education when they make which statement about anaphylaxis?

The most common cause of anaphylaxis is penicillin.

The home health nurse is assessing a client who is immunosuppressed. What is the most essential teaching for this client and the family?

The need to report any slight changes in the client's health status

The nurse assesses soft subcutaneous nodules along the line of the tendons in a patient's hand and wrist. What does this finding indicate to the nurse?

The patient has rheumatoid arthritis. Explanation: The subcutaneous nodules of rheumatoid arthritis are soft and occur within and along tendons that provide extensor function to the joints. Osteoarthritic nodules are hard and painless and represent bony overgrowth that has resulted from destruction of the cartilaginous surface of bone within the joint capsule. Lupus and neurofibromatosis are not associated with the production of nodules.

What education should the nurse provide to the patient taking long-term corticosteroids?

The patient should not stop taking the medication abruptly and should be weaned off of the medication.

A nurse is teaching a client about preventing osteoporosis. Which teaching point is correct? To prevent fractures, the client should avoid strenuous exercise. Obtaining the recommended daily allowance of calcium requires taking a calcium supplement. The recommended daily allowance of calcium may be found in a wide variety of foods. Obtaining an X-ray of the bones every 3 years is recommended to detect bone loss.

The recommended daily allowance of calcium may be found in a wide variety of foods. Explanation: Premenopausal women require 1,000 mg of calcium per day. Postmenopausal women require 1,500 mg per day. Clients usually can get the recommended daily requirement of calcium by eating a varied diet. Osteoporosis doesn't show up on ordinary X-rays until 30% of bone has been lost. Bone densitometry, however, can detect bone loss of 3% or less. This test is sometimes recommended routinely for women older than 35 who are at risk for osteoporosis. Strenuous exercise won't cause fractures. Although supplements are available, they aren't always necessary.

The nurse is caring for a patient with dark skin who is having gastrointestinal bleeding. How can the nurse determine from skin color change that shock may be present?

The skin is ashen gray and dull.

The nurse is preparing a client to have his cast cut off after having it for 6 weeks to treat a fractured tibia. What should the nurse inform the client prior to the cast being removed? The leg strength is enforced by the wearing of the cast. The leg will look as it did prior to the cast being applied. The skin may be covered with a yellowish crust that will shed in a few days. The leg will look moist and will have small bumps that will go away in a few days.

The skin may be covered with a yellowish crust that will shed in a few days. Explanation: Once the cast is off, the skin appears mottled and may be covered with a yellowish crust composed of accumulated body oil and dead skin. The client usually sheds this residue in a few days. The leg will not look as it did prior to the cast but will regain the same shape and status as the other leg. There should be no bumps underneath the cast. The leg may be weak and stiff for some time after the cast is removed, not stronger.

Identify descriptors of the pathophysiologic process seen in osteomalacia. Select all that apply. There is a deficiency of activated vitamin D (calcitriol). The bone mass is structurally weaker, and bone deformities occur. Calcium and phosphate are not moved to the bones. Excessive osteoclastic activity causes the bones to become soft and bowed initially; later, the bones thicken but are not well formed, making the bones weak and prone to fracture.

There is a deficiency of activated vitamin D (calcitriol). The bone mass is structurally weaker, and bone deformities occur. Calcium and phosphate are not moved to the bones. Explanation: In the pathophysiologic process seen in osteomalacia, there is a deficiency of activated vitamin D (calcitriol), calcium and phosphate are not moved to the bones, the bone mass is structurally weaker, and bone deformities occur.

A nurse is assessing the skin of a client who has frostbite. The client has small blisters that contain blood and the skin of the affected area does not blanch. The nurse should classify this injury as which of the following?

Third-degree frostbite When a client has third-degree frostbite, the skin of the affected area has small blisters that are blood filled and the skin does not blanch.

A client is coming to the office to have a growth removed by the doctor. The client asks "What does cryosurgery do to the growth?" What is the correct response?

Through the application of extreme cold, the tissue is destroyed.

A client arrives at the clinic and reports a very sore throat as well as a fever. A rapid strep test returns a positive result and the client is given a prescription for an antibiotic. How did the streptococcal organism gain access to the client to cause this infection?

Through the mucous membranes of the throat

A nurse is teaching a client about the causes of osteoporosis. The nurse should include which of the following types of medication therapy as a risk factor for osteoporosis?

Thyroid hormones Long-term use of synthetic thyroid hormone, such as levothyroxine, can accelerate bone loss

A client is scheduled to receive an intravenous immunoglobulin (IVIG) infusion. The client asks the nurse about the infusion's administration and its adverse effects. Which condition should the nurse instruct this client to report immediately?

Tickle in the throat

A nurse is caring for a client who has testicular cancer and is experiencing peripheral neuropathy as an adverse effect of chemotherapy. Which of the following client manifestations is an expected finding of peripheral neuropathy?

Tingling of the hands and feet Several chemotherapeutic agents might cause peripheral neuropathy. One of the major manifestations of peripheral neuropathy is numbness and tingling of an extremity.

A client that is HIV+ has been diagnosed with Pneumocystis pneumonia caused by P. jiroveci. What medication does the nurse expect that the client will take for the treatment of this infection?

Trimethoprim-sulfamethoxazole

The nurse is preparing to assess a client's integumentary status. Which charactertistics of the skin will the nurse assess by using the technique of palpation? Select all that apply. Color Turgor Edema Signs of infestations Elasticity

Turgor Edema Elasticity

A client is presenting an anaphylactic response to unknowingly ingesting nuts at a family celebration. What type of hypersensitivity did this client exhibit?

Type I

In its attempt to suppress allergic responses, the body releases several chemicals which have a role in mediating physical reactions. Epinephrine, which interferes with vasoactive chemical release from mast cells, is instrumental in suppressing which type of hypersensitivity response?

Type I

A nurse is providing discharge instructions to a client who is postoperative following a surgical exiling of a basal cell carcinoma. Which of the following findings should the nurse include as an indication of potential malignancy of a mole?

Ulceration Ulceration, bleeding, or exudate on are indications of a mole's potential malignancy. Increasing size is also a warning sign. The nurse should emphasize the importance of lifetime follow-up evaluations and the proper techniques for self-examination of the skin every month

The spleen acts as a filter for old red blood cells, holding a reserve of blood in case of hemorrhagic shock. It is also an area where lymphocytes can concentrate. It can become enlarged (splenomegaly) in certain hematologic disorders and cancers. To assess an enlarged spleen, the nurse would palpate the area of the:

Upper left quadrant of the abdomen.

A nurse in a clinic is caring for a female client who has a new diagnosis of acne vulgaris on her cheeks. Which of the following should the nurse include in the teaching plan for this client?

Use a new cosmetic pad with each limited application of makeup Use of a new cosmetic pad with each makeup application decreases the risk of reinfection. Makeup should be applied on a limited basis, as many are oil-based products, clog pores, and exacerbate acne

A client recently received lip and tongue piercings and subsequently developed a superinfection of candidiasis from the antibacterial mouthwash. What would the nurse recommend for this client?

Use an antifungal mouthwash or salt water.

The home health nurse is caring for a client with scabies. When instructing on the proper procedure to wash preworn contaminated clothing, which nursing instruction is essential?

Use hot water throughout wash cycle.

A client has undergone a kidney transplant. The nurse is concerned about a compromised immune system in this client for which reason?

Use of anti-rejection drugs

A 38-year-old client has begun to suffer from rheumatoid arthritis and is being assessed for disorders of the immune system. The client works as an aide at a facility that cares for children infected with AIDS. Which is the most important factor related to the client's assessment?

Use of other drugs

A nurse is obtaining a heath history from a client who has cancer of the cervix. Which of the following manifestations should the nurse expect?

Vaginal bleeding The most common manifestation of cancer of the cervix is painless vaginal bleeding.

The nurse reviews laboratory results requested to track HIV. What laboratory test measures HIV RNA levels and is the best predictor of HIV disease progression?

Viral load

What test will the nurse assess to determine the client's response to antiretroviral therapy?

Viral load

A nurse is caring for a client who has sa large wound healing by secondary intention. The nurse should inform the client that, in addition to protein, which of the following nutrients promotes wound healing?

Vitamin C A diet high in protein and vitamin C is recommended because these nutrients promote wound healing

Which may occur if a client experiences compartment syndrome in an upper extremity? Subluxation Callus Volkmann's contracture Whiplash injury

Volkmann's contracture Explanation: If compartment syndrome occurs in an upper extremity, it may lead to Volkmann's contracture, a claw-like deformity of the hand resulting from obstructed arterial blood flow to the forearm and hand. A whiplash injury is a cervical spine sprain. Callus refers to the healing mass that occurs with true bone formation after a fracture. Subluxation refers to a partial dislocation.

A nurse is planning discharge teaching regarding exercise for a client at risk for osteoporosis. Which exercise would the nurse be most likely to suggest?

Walking Weight-bearing exercises should be incorporated into the client's lifestyle activities. Walking is a low-impact method of weight-bearing exercise and would be the most universal or most likely form of exercise for the nurse to recommend. Bicycling, and swimming are not weight-bearing exercise and will not increase bone density. Yoga may or may not be weight-bearing exercise depending on the yoga poses being performed; it is not as likely as walking to be recommended by the nurse.

A nurse is preparing to teach a parent how to care for a child who has impetigo contagiosa. Which of the following information should the nurse plan to include in the teaching? Keep the child on droplet precautions at home Wash clothing in hot water Immunize household contacts for the disease Give the child a chlorine bath twice daily

Wash clothing in hot water The nurse should teach the parent to ensure the child changes her clothes every day and to wash all clothing in hot water

The nurse is assisting a client with removing shoes prior to an examination and observes that the client has a flexion deformity of several toes on both feet of the proximal interphalangeal (PIP) joints. What can the nurse encourage the client to do?

Wear properly fitting shoes. Explanation: Hammer toe is a flexion deformity of the PIP joint and may involve several toes and may result from wearing poorly fitting shoes. They will not straighten by binding the toes or doing active range of motion exercises. Surgery is an option but should be discussed with an orthopedic surgeon or podiatrist.

The nurse is checking the traction apparatus for a client in skin traction. Which finding would require the nurse to intervene?

Weights hanging and touching the floor Explanation: When checking traction equipment, the weights should be freely hanging. Weights that touch the floor require the nurse to intervene. The body should be aligned in an opposite line to the pull of the traction. The ropes should be freely moving over unobstructed pulleys.

A nurse is assessing a client with a new skin disorder. Which questions would the nurse include when asking the client about the change in skin condition? Select all that apply. When did the disorder first begin, and where did it first appear? Where are the lesions located? Has the problem spread? Have you tried to treat the lesions? Do you exercise daily?

When did the disorder first begin, and where did it first appear? Where are the lesions located? Has the problem spread? Have you tried to treat the lesions?

The parents of a child with contact dermatitis are asking questions about the reaction within the immune system. What description regarding contact dermatitis as a type IV hypersensitivity reaction is accurate?

a delayed-type hypersensitivity that is mediated by T cells

Crystalloid

a fluid containing soluble mineral ions and water in solution

Hypertonic solution

a solution with an osmolality higher than that of serum

Alkalosis

an acid-base imbalance characterized by a reduction in H+ concentration or increase in bicarbonate concentration (increased blood pH) (A high arterial pH with either decreased H+ ion concentration or increased bicarbonate concentration is called metabolic alkalosis; a high arterial pH due to reduced PCO2 is called respiratory alkalosis.)

Acidosis

an acid-base imbalance characterized by an increase in H+ concentration (decreased blood pH) (A low arterial pH due to increased H+ concentration or reduced bicarbonate concentration is called metabolic acidosis; a low arterial pH due to increased PCO2 is called respiratory acidosis.)

Which medication taken by the client in the previous 24 hours would be of greatest concern to the nurse caring for a client undergoing a bone biopsy?

aspirin Explanation: Aspirin has anti-clotting properties, and bone is a very vascular tissue. The client taking aspirin in close proximity to a bone biopsy is at increased risk for excessive bleeding.

A client with rheumatoid arthritis tells the nurse about experiencing mild tinnitus, gastric intolerance, and rectal bleeding. What medication does the nurse suspect is causing these side effects?

aspirin Explanation: Salicylates like aspirin may have side effects such as tinnitus, gastric intolerance and bleeding. While celecoxib, methotrexate, and hydroxychloroquine have GI upset effects, the tinnitus is unique to aspirin.

The nurse is performing a musculoskeletal assessment of a client in a nursing home who had a stroke 2 years ago and who has right-sided hemiplegia. The nurse notes that the girth of the client's right calf is 2 inches less in diameter than the left calf. The nurse attributes the decreased girth to

atrophy of right calf muscle. Explanation: Girth of an extremity may increase as a result of exercise, edema, or bleeding into the muscle. However, a client with right-sided hemiplegia is unable to use the right lower extremity. This client may experience atrophy of the muscles from lack of use, which results in a subsequent decrease in the girth of the calf muscle.

A client has been having joint pain and swelling in the left foot and is diagnosed with rheumatoid arthritis. The symptoms began suddenly without any identifiable cause, and the client has significant joint destruction. What type of disease is this considered?

autoimmune

There are a variety of problems that can become complications after a fracture. Which is described as a condition that occurs from interruption of the blood supply to the fracture fragments after which the bone tissue dies, most commonly in the femoral head? avascular necrosis pulmonary embolism shock fat embolism

avascular necrosis Explanation: Avascular necrosis is described as a condition that occurs from interruption of the blood supply to the fracture fragments after which the bone tissue dies, most commonly in the femoral head.

A client has begun sensitivity testing to determine the allergen which caused an anaphylactic reaction 3 weeks ago. In scratch testing, which part of the body is more sensitive to allergens?

back

Students are reviewing the cycle of hair growth in people, identifying that rate of hair growth varies on different parts of the body. The students demonstrate understanding of this information when they identify which area as having the most rapid rate?

beard

A client with AIDS has been tested for cytomegalovirus (CMV) with positive titers. What severe complication should the nurse be alert for with cytomegalovirus?

blindness

The nurse practitioner who is monitoring the patient's progression of HIV is aware that the most debilitating gastrointestinal condition found in up to 90% of all AIDS patients is:

chronic diarrhea

The nurse is reviewing the medication administration record of the client. Which medication would lead the nurse to suspect that the client is suffering from an acute attack of gout?

colchicine Explanation: Colchicine is prescribed for the treatment of an acute attack of gout.

The nurse is working with community groups. At which of the following locations would the nurse anticipate a possible scabies outbreak?

college dormitory

The nurse is applying a cool compress to the forehead of a client with an elevated temperature. This is an example of what type of heat loss?

conduction

The nurse is assessing the feet of a patient and observes an overgrowth of the horny layer of the epidermis. What does the nurse recognize this condition as?

corn A corn is an area of hyperkeratosis (overgrowth of a horny layer of epidermis) produced by internal pressure (the underlying bone is prominent because of a congenital or acquired abnormality, commonly arthritis) or external pressure (ill-fitting shoes). The fifth toe is most frequently involved, but any toe may be involved.

The nurse is conducting an admission history and physical examination of a client with a history of contact dermatitis. The nurse assesses whether the client uses which medication classification?

corticosteroids

Which is not one of the general nursing measures employed when caring for the client with a fracture?

cranial nerve assessment Explanation: Cranial nerve assessment would only be carried out for head-related injuries or diseases. General nursing measures include administering analgesics, providing comfort measures, assisting with ADLs, preventing constipation, promoting physical mobility, preventing infection, maintaining skin integrity, and preparing client for self-care.

The purpose of melanin is to:

determine skin color

A client who has fallen and injured a hip cannot place weight on the leg and is in significant pain. After radiographs indicate intact but malpositioned bones, what would the physician diagnose? fracture dislocation strain sprain

dislocation Explanation: In joint dislocation, radiographic films show intact yet malpositioned bones. Arthrography or arthroscopy may reveal damage to other structures in the joint capsule. A strain is an injury to a muscle when it is stretched or pulled beyond its capacity. Sprains are injuries to the ligaments surrounding a joint. A fracture is a break in the continuity of a bone.

The nurse is gathering a health history for a client with osteoarthritis. What clinical manifestation will the nurse expect to find?

early morning stiffness Osteoarthritis is characterized by early morning stiffness that decreases with activity. Large joints are usually involved with osteoarthritis. Joint pain is a constant with osteoarthritis. Clients with rheumatoid arthritis have subcutaneous nodules.

A client is being treated for hyperuricemia. Part of the treatment strategy is for the client to avoid contributing factors whenever possible. Which activities might bring on an acute attack?

eating organ meats and sardines During an acute attack, high-purine foods are avoided, including organ meats, gravies, meat extracts, anchovies, herring, mackerel, sardines, and scallops. The other listed factors do not worsen attacks.

A nurse is caring for a client experiencing an exacerbation of plaque psoriasis. The nurse assesses the area and documents a proliferation of which cell type?

epidermal

Which of the following was formerly called a bunion? Morton's neuroma Hallux valgus Ganglion Plantar fasciitis

hallux valgus Hallux valgus (commonly called a bunion) is a deformity in which the great toe deviates laterally. Plantar fasciitis, an inflammation of the foot-supporting fascia, present as an acute onset of heel pain experienced with the first steps in the morning. Morton's neuroma is a swelling of the third (lateral) branch of the median plantar nerve. A ganglion, a collection of gelatinous material near the tendon sheaths and joints, appears as a round, firm compressible cystic swelling, usually on the dorsum of the wrist.

A client has come to the clinic with foot pain. The physician has described the client's condition as a flexion deformity of the proximal interphalangeal joint. What is the name of this disorder?

hammer toe Hammer toe is a flexion deformity of the proximal interphalangeal joint. Mallet toe is a flexion deformity of the distal interphalangeal joint. Bunion is a deformity of the great toe at its metatarsophalangeal joint. Heberden's nodes are bony enlargements of the distal interphalangeal joints.

A nurse in an ophthalmology clinic is interviewing a client who was referred by his primary care provider for suspicion of cataracts. The nurse should expect the client to report

having a decreased ability to perceive colors Symptoms of cataracts include painless blurred vision and a decrease in the ability to perceive colors.

When describing the effects of treatment for phagocytic cell disorders, which of the following would the nurse include as being the most successful?

hematopoietic stem cell transplantation

The nurse is assessing the integumentary system of a client with Cushing syndrome. The nurse anticipates which finding?

hirsutism

Which body substance causes increased gastric secretion, dilation of capillaries, and constriction of the bronchial smooth muscle?

histamine

Which is a primary chemical mediator of hypersensitivity?

histamine

The classic lesions of impetigo manifest as

honey-yellow crusted lesions on an erythematous base.

A client is prescribed a disease-modifying antirheumatic drug that is successful in the treatment of rheumatoid arthritis but has side effects, including retinal eye changes. What medication will the nurse anticipate educating the client about?

hydroxychloroquine Explanation: The DMARD hydroxychloroquine is associated with visual changes, GI upset, skin rash, headaches, photosensitivity, and bleaching of hair. The nurse should emphasize the need for ophthalmologic examinations every 6-12 months. Azathioprine, diclofenac, and cyclophosphamide do not have visual changes as a side effect

Which layer of the skin is made of primarily adipose tissue?

hypodermis

Production of melanin is controlled by a hormone secreted by which gland?

hypothalamus

A client has been in a motor vehicle collision. Radiographs indicate a fractured humerus; the client is awaiting the casting of the upper extremity and admission to the orthopedic unit. What is the primary treatment for musculoskeletal trauma? enhancing complications immobilization external rotation surgical repair

immobilization Explanation: Treatment of musculoskeletal trauma involves immobilization of the injured area until it has healed.

A client with severe combined immunodeficiency is to receive a hematopoietic stem cell transplant. What would the nurse expect to be started?

immunosuppressive agents

The nurse is discussing life management with the client with rheumatoid arthritis in a health clinic. What assessment finding indicates the client is having difficulty implementing self-care?

increased fatigue Explanation: Fatigue is common with rheumatoid arthritis. Finding a balance between activity and rest is an essential part of the therapeutic regimen. The client is reporting being able to do ADLs and decreased joint pain. The client's weight gain of 2 pounds does not correlate with self-care problems.

The body has several mechanisms to fight disease, one of which is sending chemical messengers. The messengers released by lymphocytes, monocytes, and macrophages have differing roles in the immune response. Which messenger enables cells to resist viral replication and slow viral replication?

interferons

During a routine assessment of a client, the nurse notes that the client's nails are concave. Which condition is indicated by this finding?

iron deficiency anemia

A nurse discovers scabies when assessing a client who has just been transferred to the medical-surgical unit from the day surgery unit. To prevent scabies infection in other clients, the nurse should:

isolate the client's bed linens until the client is no longer infectious.

A patient has a serum bilirubin concentration of 3 mg/100 mL. What does the nurse observe when performing a skin assessment on this patient?

jaundice

A client has an exaggerated convex curvature of the thoracic spine. What is this condition called?

kyphosis Explanation: Kyphosis is an exaggerated convex curvature of the thoracic spine. Lordosis is an excessive concave curvature of the lumbar spine. Scoliosis is a lateral curvature of the spine. Diaphyses are the long shafts of bones in the arms and legs.

The nurse is assessing a client with a history of ankylosing spondylitis. What will the nurse most commonly assess?

low back pain Explanation: The most common symptoms of ankylosing spondylitis are low back pain and stiffness. A red, butterfly-shaped rash on the face and a patchy loss of hair are associated with systemic lupus erythematosus. Ankylosing spondylitis does not affect urine output.

Which of the following could be a possible cause of cyanosis?

low tissue oxygenation

Homeostasis

maintenance of a constant internal equilibrium in a biologic system

Nursing care for the client with fibromyalgia should be guided by the assumption that patients with fibromyalgia

may feel as if their symptoms are not taken seriously. Because clients present with widespread symptoms that are often vague in nature, health care providers may misdiagnose them. Clients feel as though people are not listening to them. Nurses need to provide support and encouragement. Symptoms of disease vary from client to client and respond to different treatments. Clients do not lose their ability to walk.

While assessing the skin of a 45-year-old, fair-skinned female client, the nurse notes a lesion on the medial aspect of her lower leg. It has irregular borders, with various shades of black and brown. The client states that the lesion itches occasionally and bled slightly a few weeks ago. She also reveals a history of sunburns. Based on these signs and symptoms, the nurse suspects:

melanoma

A client with rheumatoid arthritis has experienced increasing pain and progressing inflammation of the hands and feet. What would be the expected goal of the likely prescribed treatment regimen?

minimizing damage Explanation: Although RA cannot be cured, much can be done to minimize damage. Treatment goals include decreasing joint inflammation before bony ankylosis occurs, relieving discomfort, preventing or correcting deformities, and maintaining or restoring function of affected structures. Early treatment leads to the best results.

The nurse is caring for a 24-year-old female client with a right tibial fracture treated with a cast 2 hours ago. The client now reports unrelenting pain, rated as 7/10, despite taking oxycodone, and decreased sensation in the right foot. A nursing assessment reveals the right foot is cooler and paler than the left foot, with delayed capillary refill and a weak pulse. Based on the nursing assessment, the priority action the nurse should take is to

notify the orthopedic health care provider immediately bivalving of the cast

A fracture is considered pathologic when it involves damage to the skin or mucous membranes. results in a fragment of bone being pulled away by a ligament or tendon and its attachment. presents as one side of the bone being broken and the other side being bent. occurs through an area of diseased bone.

occurs through an area of diseased bone. Explanation: Pathologic fractures can occur without the trauma of a fall. An avulsion fracture results in a fragment of bone being pulled away by a ligament or tendon and its attachment. A greenstick fracture presents as one side of the bone being broken and the other side being bent. A compound fracture involves damage to the skin or mucous membranes.

A fracture is considered pathologic when it involves damage to the skin or mucous membranes. results in a fragment of bone being pulled away by a ligament or tendon and its attachment. occurs through an area of diseased bone. presents as one side of the bone being broken and the other side being bent.

occurs through an area of diseased bone. Explanation: Pathologic fractures can occur without the trauma of a fall. An avulsion fracture results in a fragment of bone being pulled away by a ligament or tendon and its attachment. A greenstick fracture presents as one side of the bone being broken and the other side being bent. A compound fracture involves damage to the skin or mucous membranes.

The nurse is performing a health history with a new client in the clinic. What is the most common reason for a client to seek medical attention for arthritis?

pain Explanation: The symptom that most commonly causes a person to seek medical attention is pain. Other common symptoms include joint swelling, limited movement, stiffness, weakness, and fatigue.

A nurse knows that more than 50% of clients with CVID develop the following disorder.

pernicious anemia

The nurse notes several very small, round, red and purple macules on a patient's skin. The patient has a history of anticoagulant use. The nurse records this finding as which of the following?

petechiae

The nurse working the medical-surgical floor knows that the nitroblue tetrazolium reductase (NTR) test is used to diagnose immunodeficiency disorders related to

phagocytic cells

The nurse is teaching a client about contact dermatitis. What type of contact dermatitis requires light exposure in addition to allergen contact?

photoallergic

Active transport

physiologic pump that uses energy to move fluid or electrolytes from one region to another

The nurse is teaching a client about allergic rhinitis and its triggers. What is the most common trigger for the respiratory allergic response?

plant pollen

A nurse is admitting a client with toxic epidermal necrolysis. What is the nursing priority in preventing sepsis?

preventing infection

Which intervention is the single most important aspect for the client at risk for anaphylaxis?

prevention

A client has a boil that is located in the left axillary area and is elevated with a raised border, and filled with pus. How would the nurse document this type of lesion?

pustule

A client with HIV will be started on a medication regimen of three medications. What class of drugs will the nurse instruct the client about?

reverse transcriptase inhibitors

A client comes to the clinic reporting nasal congestion and states, "I've been using an over-the-counter nasal spray that seemed to help at first, but then I got even more congested than before I started the medication. I continued the nasal spray, but it seems to be worse." What reaction will the nurse suspect?

rhinitis medicamentosa

The nurse is completing the intake assessment of a client new to the allergy clinic. The client states that he was taking nose drops six times a day to relieve his nasal congestion. The nasal congestion increased, causing him to increase his usage of the nasal spray to eight times a day. But again the congestion worsened. The nurse communicates to the health care provider that the client experienced

rhinitis medicamentosa

A client with psoriasis visits the dermatology clinic. When inspecting the affected areas, the nurse expects to see which type of secondary lesion?

scale

A patient asks the nurse if it would be all right to take an over-the-counter antihistamine for the treatment of a rash. What should the nurse educate the patient is a major side effect of antihistamines?

sedation

The nurse is talking with a group of teens about transmission of human immunodeficiency virus (HIV). What body fluids does the nurse inform them will transmit the virus? Select all that apply. semen urine breast milk blood vaginal secretions

semen breast milk blood vaginal secretions

A client has been diagnosed with a muscle strain. What does the physician mean by the term "strain"?

stretched or pulled beyond its capacity Explanation: A strain is an injury to a muscle when it is stretched or pulled beyond its capacity.

The client asks the nurse about types of exercise that do not stress the joints. What exercise will the nurse include in the teaching plan?

t'ai chi Explanation: T'ai chi is low impact, so this is the best exercise for low joint impact. Jogging, weight lifting, and running on a treadmill are high-impact, jarring types of exercise.

osmolarity

the number of milliosmoles (the standard unit of osmotic pressure) per liter of solution; expressed as milliosmoles per liter (mOsm/L); describes the concentration of solutes or dissolved particles

osmosis

the process by which fluid moves across a semipermeable membrane from an area of low solute concentration to an area of high solute concentration; the process continues until the solute concentrations are equal on both sides of the membrane

Diffusion

the process by which solutes move from an area of higher concentration to one of lower concentration; does not require expenditure of energy

Petechiae are associated with which of the following disorders?

thrombocytopenia

A client is informed of having a low white blood cell count and that the client is at risk for the development of infections. The client asks, "Where do I make new white blood cells?" What is the best response by the nurse?

"White blood cells are produced in the bone marrow."

Which medication classification is known to inhibit prostaglandin synthesis or release?

Nonsteroidal anti-inflammatory drugs (in large doses)

A nurse is collecting a heath history from a female client who is undergoing screening for breast cancer. Which of the following factors should the nurse identify for placing the client at the greatest risk for developing breast cancer?

Over 50 years of age A female client whose age is over 50 years has a high increased risk for developing breast cancer

A client in a late stage of acquired immunodeficiency syndrome (AIDS) shows signs of AIDS-related dementia. Which nursing diagnosis takes highest priority?

Risk for injury

When teaching a client how to prevent low back pain as a result of lifting, the nurse should instruct the client to:

avoid overreaching. Explanation: Instructions for the client with low back pain should include that, when lifting, the client should avoid overreaching. The client should also keep the load close to the body, bend the knees and tighten the abdominal muscles, use a wide base of support, and use a back brace to protect the back.

A client is diagnosed with a fracture of a diarthrosis joint. What is an example of this type of joint?

Elbow Explanation: A diarthrosis joint, like the elbow, is freely movable. The skull is an example of an immovable joint. The vertebral joints and symphysis pubis are amphiarthrosis joints that have limited motion

A patient is diagnosed with osteogenic sarcoma. What laboratory studies should the nurse monitor for the presence of elevation?

Alkaline phosphatase Explanation: Serum alkaline phosphatase levels are frequently elevated with osteogenic sarcoma or bone metastasis. Hypercalcemia is also present with bone metastases from breast, lung, or kidney cancer. Symptoms of hypercalcemia include muscle weakness, fatigue, anorexia, nausea, vomiting, polyuria, cardiac dysrhythmias, seizures, and coma. Hypercalcemia must be identified and treated promptly.

A client has been diagnosed with shingles. Which of the following medication classifications will reduce the severity and prevent development of new lesions?

Antiviral

Activation of a natural immunity response is enhanced by physical and chemical barriers. Which of the following is a physical barrier, which the nurse knows can be altered by illness, nutrition, or lifestyle?

Cilia of the respiratory tract

A patient is being evaluated for nodular cystic acne. What systemic pharmacologic agent may be prescribed for the treatment of this disorder?

Isotretinoin (Accutane)

A nurse is examining a client's hair and notes that the scalp is within normal limits. What characteristics will the nurse most likely document? Select all that apply. Smooth Intact Free of lesions Scales Flaking

Smooth Intact Free of lesions

Which microorganism is known to cause retinitis in people with HIV/AIDS?

cytomegalovirus

The nurse notes that an older adult was treated for a wound infection and pneumonia within the last 6 months. Which factor will the nurse attribute to this client's illnesses?

Immunosenescence

The development of a positive HIV antibody test following initial infection generally occurs in which timeframe?

4 weeks

Normal magnesium concentration

1.8-3.6 mg/dL

Which characteristic has NOT been implicated as a factor for noncompliance with antiretroviral treatment?

Past substance abuse

Which immunoglobulin assumes a major role in bloodborne and tissue infections?

IgG

Photochemotherapy combines the use of ultraviolet A (UVA) and which of the following medications?

Psoralen

A patient is scheduled for Mohs microscopic surgery for removal of a skin cancer lesion on his forehead. The nurse knows to prepare the patient by explaining that this type of surgery requires:

Removal of the tumor, layer by layer.

A nurse is examining a client's scalp for evidence of lice. The nurse should pay particular attention to which part of the scalp?

behind the ears

What is the most common cause of anaphylaxis?

penicillin


Ensembles d'études connexes

Unit 6: Classical Genetics OLI Quizzes

View Set

PHRM 829 - Most Missed Exam Questions

View Set

Marketing 351 Ch. 1,2,5,7 Exam 1 PHAM

View Set

Comptia 220-801 7.1.11 Practice Test Questions

View Set